Sie sind auf Seite 1von 101

1

Lung Pathology
NORMALS FOR REFERENCE

1. Normal lung: A lung is bisected, revealing branching conducting airways (bronchi) and spongy brown parenchyma.

4. Normal alveolar parenchyma: Terminal and respiratory bronchioles (*), alveolar ducts, alveolar sacs, and a small artery are shown.

5. Normal alveolar parenchyma: A pulmonary macrophage (arrow) lies in the central alveolus.

2 and 3. Normal bronchus: The bronchial wall includes the respiratory mucosa, smooth muscle, submucosa with seromucinous glands, and cartilage. The epithelium is pseudostratified and columnar, and consists of ciliated cells, goblet cells and reserve cells. Neuroendocrine (Kulchitsky) cells cannot be distinguished.

6. Normal interlobular septum with lymphatic: A lymphatic (*) courses through an interlobular septum.

ATELECTASIS
Definition: incomplete expansion of the lung, or collapse of previously inflated lung tissue Atelectatic parenchyma is predisposed to infection Depending upon the amount of lung tissue involved, oxygenation may be reduced Usually reversible - parenchyma can be reexpanded unless scarring has occurred

Etiologies/Types of Atelectasis 1. Obstructive (absorptive) - complete airway obstruction ----> absorption of air trapped distally ----> collapse (mediastinum may shift toward the collapsed lung) Causes: Excessive bronchial secretions and mucus plugging (asthma, chronic bronchitis, postoperative)

Masses (aspirated foreign bodies, tumors)

2. Compressive - compression of the lung by a substance in the pleural space [air (pneumothorax), blood (hemothorax), fluid (effusion)] or an abnormally elevated diaphragm (basal atelectasis) associated with peritonitis or subdiaphragmatic abscess; mediastinum may shift away from the collapsed lung 3. Contraction - collapse of parenchyma around fibrotic areas of lung or pleura. So-called round atelectasis is a type of contraction atelectasis: the visceral pleura develops a localized area of fibrosis associated with an infolding of the pleura that extends downward into the underlying lung tissue. Many patients have a history of asbestos exposure. Significance: round atelectasis may look like a mass radiographically. 4. Patchy - with loss of or inadequate surfactant (ARDS, hyaline membrane disease of newborn)

DISEASES OF VASCULAR ORIGIN


Pulmonary edema
Lungs are heavy, with increased fluid particularly in the lower lobe bases Histologically, there is eosinophilic fluid in alveoli, congested capillaries, focal intraalveolar hemorrhages and hemosiderin-laden macrophages ("heart failure cells") Causes include hemodynamic alterations and microvascular injury

Hemodynamic causes: processes that alter the balance between forces moving fluid into and out of capillaries (Starlings law of capillary interstitial fluid exchange) Increased hydrostatic pressure - left-sided heart failure, fluid overload, pulmonary venous obstruction Decreased oncotic pressure - hypoalbuminemia (nephrotic syndrome, cirrhosis, protein-losing enteropathy) Lymphatic obstruction - tumor blockage of lymphatics Microvascular injury - injury to the alveolar capillary endothelium ----> fluids and proteins leak into the interstitium and then the alveoli; can be localized (ex: near a pneumonia) or diffuse (see section on ARDS)

7. Pulmonary edema: Eosinophilic fluid fills alveoli, and focal hemorrhages are also present.

8. Chronic passive congestion: Hemosiderin-laden macrophages and edema fluid occupy alveoli.

Chronic passive congestion


With longstanding congestion of the lungs, such as is seen with mitral stenosis, numerous hemosiderin-laden macrophages collect in alveoli and interstitial fibrosis develops; lungs look brown and feel firm (brown induration)

Pulmonary embolism, hemorrhage, and infarction


General facts:

Pulmonary emboli cause about 50,000 deaths/yr. in the U.S. Most (>95%) of the emboli arise in the large lower leg veins, and occasionally large pelvic veins or the right side of the heart are the source of the emboli Increased incidence in patients with o Primary hypercoagulable states: factor V Leiden, antithrombin III or protein C deficiency, antiphospholipid syndrome, prothrombin 20210 A, others o Secondary hypercoagulable states: chronic immobilization, late pregnancy, oral contraceptive use, cancer, recent surgery

Embolism is much more common than thrombosis in pulmonary arteries; thrombosis usually requires a predisposing condition such as tumor or infection Prevention: early ambulation of post-surgical and postpartum patients, elastic stockings and leg exercises for bedridden people, anticoagulation in high risk people, or use of a filter in the inferior vena cava

Emboli lead to respiratory compromise due to V/Q mismatch, and hemodynamic compromise caused by increased resistance to blood flow induced by obstruction. The clinical significance of an embolus depends on: Extent of obstruction of pulmonary artery blood flow (size, # of vessels occluded) Collateral (bronchial) circulation Underlying cardiorespiratory status of the patient

OUTCOME Nothing (clinically silent) Sudden death / Acute right heart failure / Electromechanical dissociation Infarction (visible on CXR 12 36 hours after embolism occurs) Hemorrhage Pulmonary hypertension

MECHANISM

FREQ. 60-80%

>60% of the pulmonary vasculature is obstructed Inadequate collateral pulmonary blood flow; often coexisting heart or lung disease is present Collateral blood flow is sufficient to sustain viability of tissues Usually multiple emboli over time; can cause R ventricular hypertrophy

5% 10-15%

10-15% 2-3%

Morphology: Embolus: red cells and fibrin fill arterial lumen, adhering to the endothelial surface ----> may lyse, contract, organize and/or recanalize Hemorrhage: blood in alveoli and interstitium, with preservation of architecture Infarct: usually wedge-shaped and peripheral, with embolus at apex acute - ischemic necrosis of the parenchyma with hemorrhage

older - scar (fibrosis), often with hemosiderin-laden macrophages

9. Pulmonary embolus: Blood clot fills the lumen of the pulmonary artery.

10. Recent pulmonary infarct: The infarct lies in a typical location (subpleural) and has the typical wedge shape. The hemorrhagic appearance tells you that it is a recent (not old) infarct.

11-12. Organizing pulmonary embolus and infarct: The lung tissue shows coagulative necrosis and hemorrhage. A small artery is filled by an organizing fibrin embolus.

Special types of emboli: Embolic carcinomatosis - some carcinomas of the breast, stomach, and lung massively invade blood vessels, widely disseminate and block pulmonary arteries Septic embolism - fragments of an infected (bacteria or fungi) venous thrombus or tricuspid valve vegetation embolize, sometimes producing septic infarcts or thrombarteritis; microscopically - neutrophils and organisms Fat embolism - from trauma, esp. fractures or extensive subcutaneous tissue injury, or surgery involving bones Air embolism - from trauma, surgery, intravenous injection, obstetrical procedures; usually takes > 100 cc to have a clinical effect via occlusion of vessels Bone marrow embolism - after CPR with chest massage leading to rib fractures

Amniotic fluid embolism - around the time of delivery; can cause shock and ARDS Starch or talc particles - contained in drug solutions injected by addicts Parasites - Dirofilaria immitis (dog heart worm)

13. Tumor emboli: A plug of metastatic carcinoma and fibrin fills a small pulmonary artery. The parenchyma at left is infarcted.

14. Intravenous talcosis: Abundant talc (polarizable) is present adjacent to blood vessels, in this lung from an intravenous drug abuser.

15. Dirofilaria immitis: The body of a dog heartworm is curled in the lumen of a small pulmonary artery (Masson trichrome stain).

Pulmonary hypertension: mean pulmonary pressure >or = 1/4 X systemic blood


pressure (normally, pulmonary blood pressure is about one-eighth of systemic blood pressure). Most cases are secondary to structural cardiopulmonary conditions that increase pulmonary blood flow and/or pressure, pulmonary vascular resistance (PVR), or left heart resistance to blood flow. A smaller number of cases are primary.

Simonneau G, Gali N, Rubin LJ, et al. Clinical classifi cation of pulmonary hypertension. J Am Coll Cardiol. 2004;43 (12 Suppl S):5S-12S

SECONDARY CAUSES Chronic obstructive and interstitial lung diseases Recurrent pulmonary emboli Congenital heart diseases with left to right shunts Acquired heart diseases (ex: mitral stenosis) Collagen vascular disorders

MECHANISMS 1. Chronic hypoxia ---> vasoconstriction ---> inc PVR 2. Decreased numbers of alveolar capillaries Decrease in cross-sectional area of open pulmonary vessels Increased pulmonary blood flow ---> inc PVR Increased left atrial pressure ---> increased pulmonary venous pressure ----> increased pulmonary arterial pressure Vascular inflammation and fibrosis

Pathogenesis: Endothelial cell injury resulting from mechanical forces (increased blood flow and pressure), biochemical alterations (such as due to the fibrin in thromboemboli), or unknown sources (idiopathic) produces changes in the chemical microenvironment that initiate the vascular responses causing pulmonary hypertension: Vasoconstriction Promoted by dec prostacyclin and nitric oxide, inc endothelin Platelet adhesion and activation - Promoted by dec prostacyclin and nitric oxide Fibrin formation and deposition Migration and proliferation of vascular smooth muscle cells and production of extracellular matrix Vasospasm Primary (idiopathic) pulmonary hypertension: Many cases are linked to mutations in the bone morphogenetic protein receptor type 2 (BMPR2) signaling pathway - BMPR2 locus in on chromosome 2q33; signaling is important for inhibiting vascular smooth muscle cell proliferation and favoring apoptosis - Since the disease only occurs in 10-20% of people with BMPR2 mutations, other modifier genes and/or environmental triggers may be needed to cause clinical disease Occasional familial cases (autosomal dominant) Usually fatal within 2-5 yrs. from cor pulmonale, though vasodilators, antithrombotic medications and lung or heart-lung transplantation may prolong life Morphology (pulmonary hypertensive disorders): Medial hypertrophy, intimal proliferation and fibrosis in arterioles and small arteries Atheromatous deposits in main elastic arteries Capillary tufts in lumina of arteries (plexiform lesions) - indicates severe and irreversible pulmonary hypertension; more common in primary pulmonary hypertension and congenital heart disease with L-to-R shunts Additional findings specific for a given etiology: - antecedent pulmonary emboli: may see organizing/ed emboli

10

- coexisting diffuse pulmonary fibrosis, severe emphysema or chronic bronchitis

16. Pulmonary hypertension: This small artery demonstrates medial hypertrophy.

17. Pulmonary hypertension: This plexiform lesion is an aggregate of small capillaries, and indicates the existence of severe pulmonary hypertension.

Veno-occlusive disease: Small veins become occluded by fibroblasts and


collagen, leading to chronically impaired venous outflow and histologic changes of chronic passive congestion. Chemotherapeutic agents or other toxins sometimes cause this disorder, but no etiology can be determined in a large number of cases. Occurs in children and young adults. Can be treated by lung transplantation.

18. Pulmonary veno-occlusive disease: A small vein is occluded by fibrous tissue, and the surrounding lung shows changes of chronic passive congestion

Disorders associated with diffuse pulmonary hemorrhage:


With vasculitis: o Goodpasture syndrome o Antineutrophil cytoplasmic antibody (ANCA)-associated vasculitides Wegeners granulomatosis Microscopic polyangiitis Churg-Strauss syndrome o Collagen vascular diseases Systemic lupus erythematosus

11

Others o Behcet disease Without vasculitis: o Idiopathic pulmonary hemosiderosis o Associated with coagulopathy o Associated with infections o Associated with diffuse alveolar damage o Associated with chronic passive congestion o Associated with arteriovenous malformation

Goodpasture Syndrome (Anti-glomerular basement membrane disease) Necrotizing hemorrhagic interstitial pneumonitis and glomerulonephritis caused by antibodies to a glomerular basement membrane antigen (a component of the noncollagenous domain of the alpha3 chain of collagen type IV) that cross-react with alveolar basement membranes M > F, especially in second and third decades; present with hemoptysis, consolidation, renal failure Cofactors - smoking, exposure to hydrocarbons (dry cleaning work), viral infection; associated with HLA-DR2 Morphology: intraalveolar hemorrhage +/- hemosiderin-laden macrophages; immunofluorescence (IF) reveals linear deposits of IgG along basement membranes of alveolar septa and glomeruli Treatment - plasma exchange, immunosuppression
19. Goodpastures syndrome: IF staining for IgG reveals the diagnostic linear staining along the basement membrane.

Wegeners Granulomatosis An autoimmune disease that classically involves the triad of the lungs, upper respiratory tract and kidneys, but can involve almost any anatomic site Associated with antineutrophil cytoplasmic antibodies (ANCA), which are believed to play a role in the pathogenesis of the disease Pathologic findings include vasculitis (any size vessel), necrosis, granulomatous inflammation

12

20. Wegeners granulomatosis small vessel vasculitis: The capillaries are densely infiltrated by neutrophils and there is a background of pulmonary hemorrhage.

21. Wegeners granulomatosis granuloma: This small granuloma has a central microabscess.

13

Acute respiratory distress syndrome (ARDS)


Clinical presentation - a syndrome that manifests acutely, usually within hours of the triggering event, with severe life-threatening respiratory insufficiency, cyanosis, and severe hypoxemia that is refractory to oxygen therapy, and often leads to multiorgan failure; CXR - diffuse bilateral infiltrates ("white out") mechanical ventilation is used to try to maintain oxygenation; often, high oxygen concentrations are needed, which may add to injury due to oxygen toxicity mortality rate has been 40-60%, but there has been recent improvement Triggers can be systemic inflammatory conditions or primary lung diseases or injuries, or combinations. Etiologies include
Diffuse pulmonary infections*** Inhalation of smoke, irritant gases Aspiration of gastric contents*** Sepsis*** Shock*** Trauma*** Massive air, fat, or amniotic fluid embolism Drugs - chemotherapeutic agents, narcotic (esp. heroin) overdose, occasionally other drugs Radiation Disseminated intravascular coagulation and other blood disorders Hemodialysis and cardiopulmonary bypass Uremia, diabetic ketoacidosis, and other metabolic disorders Extensive burns Acute pancreatitis Near-drowning Oxygen toxicity Pulmonary contusions

*** most common Pathogenesis: diffuse damage to the alveolar wall (capillary endothelium and alveolar epithelium) predictable series of physiologic and morphologic events that are generally similar regardless of the initiating agent Although endothelial and epithelial injury is directly caused by some of the initiating agents, native inflammatory cells and mediators are usually central to the evolution of the process. Pulmonary intravascular neutrophil aggregation occurs in many of the conditions leading to ARDS and is responsible for damage to endothelial and epithelial cells through release of toxic oxidants and proteases. ARDS, however, can also develop in a setting of neutropenia. Cytokine (TNF-, IL-1, IL-8) release from lung macrophages enhances endothelial expression of adhesion molecules for leukocytes and promotes the inflammatory response. NF-B is an important participant in transcriptional mechanisms that control expression of proinflammatory genes. Neutrophils oxygen-derived free radicals and proteases Macrophages oxygen-derived free radicals, proteases, arachidonic acid metabolites, platelet-activating factor

14

Oxygen-derived free radicals endothelial and epithelial injury increased vascular permeability and epithelial sloughing Lysosomal enzymes (proteases) destroy structural proteins (collagen, elastin) Thromboxane vasoconstriction Macrophage-derived factors transforming growth factor and platelet-derived growth factor fibroblast proliferation, collagen deposition Morphology: diffuse alveolar damage (DAD) is the pathologic lesion that occurs in victims of ARDS; it is divided into an acute (exudative) stage which occurs in the first week following injury, and an organizing (proliferative) stage, which begins about 7 days post injury; hyaline membrane disease of the newborn has very similar morphology, but the pathogenesis is different (deficiency of surfactant) PMN, macrophage activation Edema, pneumocyte necrosis, hyaline membranes, atelectasis V/Q mismatch hypoxemia

Trigger

Endothelial & epithelial injury

Acute Stage

Variable recovery of respiratory function

Fibrosis and architectural remodeling

Type II pneumocyte proliferation, interstitial inflammation and Stage fibroblast proliferation

Organizing

22. Diffuse alveolar damage/acute respiratory distress syndrome: Hyaline membranes line alveolar septa. Hyaline membranes are composed of fibrin-rich edema fluid mixed with cytoplasmic remnants from necrotic cells.

15

BACTERIAL INFECTIONS OF THE LUNG


PREDISPOSING FACTORS Impairment of pulmonary defense mechanisms or generalized lowering of host resistance (chronic disease, immunocompromise, chemotherapy, previous severe infection - particularly pneumonia) predispose to bacterial pneumonia. Loss or suppression of the cough reflex may lead to aspiration of oropharyngeal and/or gastric materials - ex: anesthesia, coma, drugs, neuromuscular disorders. Mucociliary apparatus dysfunction leads to impaired bacterial clearance. Agents and disorders that can cause, or be associated with, mucociliary apparatus dysfunction include cigarette smoke, viral infections, immotile cilia syndrome, inhalation of toxic gases, aspiration of toxic substances. Disturbance of the phagocytic or bactericidal actions of alveolar macrophages can reduce bacterial clearance. Causes include cigarette smoke, alcohol, anoxia. Hypogammaglobulinemia.

In addition, pulmonary edema and congestion (congestive heart failure) and accumulated secretions (cystic fibrosis, bronchial obstruction) can foster bacterial growth. HOW THESE INFECTIONS DEVELOP Although inhalation of bacteria is the usual mode of infection, occasionally hematogenous spread from other foci of infection can occur. Hematogenous pneumonia is a particular problem in hospitalized patients with intravenous lines. These lines can become colonized by bacteria, especially Pseudomonas aeruginosa and Staphylococcus aureus, and the organisms can embolize to the lungs where they cause pneumonias and abscesses. Nosocomial pneumonias, developing via inhalation or hematogenous routes, are not uncommon, and can be difficult to treat due to the frequency of antibiotic resistance in organisms dwelling within the hospital environment. Intubation also increases the likelihood of nosocomial infection. Bacterial contamination of respirator apparatus may occur, leading to tracheobronchial colonization, tracheobronchitis, and pneumonia.

16

BACTERIAL PNEUMONIA
Symptoms: fever, chills, cough, expectoration, expiratory rales, shortness of breath, cyanosis (if decreased vital capacity), pleuritic chest pain Course depends upon the virulence of the responsible agent, the extent of pneumonic involvement, the patient's underlying condition, and the effectiveness and promptness of antibiotic therapy; mortality < 10% Complications: lung abscess, empyema, suppurative pericarditis, bacteremia with metastatic abscess formation, extensive pulmonary fibrosis

Bronchopneumonia: patchy consolidation of one or multiple


lobes of the lung, usually representing extension of a bronchitis Very common, esp. in older people with chronic illnesses The usual organisms: o Community-acquired pneumonia: Streptococcus pneumoniae, H. influenzae, Moraxella catarrhalis, Pseudomonas aeruginosa, Staphylococcus aureus, Klebsiella pneumoniae o Nosocomial pneumonia: Pseudomonas aeruginosa, Staphylococcus aureus, Klebsiella pneumoniae, Serratia marcescens, E. coli

23. Acute bronchopneumonia: The tan-yellow spots centered around airways are foci of acute bronchopneumonia, and represent acute inflammatory exudate in bronchi, bronchioles and adjacent alveoli.

2425. Acute bronchopneumonia: Neutrophils fill a bronchiole and adjacent alveoli.

Morphology: Gross: firm gray/red/yellow foci separated by more normal appearing parenchyma

17

Microscopic: neutrophils fill bronchi, bronchioles, and adjacent alveolar spaces; necrosis may be present with more virulent organisms; with time, the exudate may resolve, leaving little fibrosis behind, or organize with variable fibrosis

Lobar Pneumonia: bacterial infection involving an entire lobe or a large portion of a


lobe Less common than in the past, due to effective antibiotic therapy which can abort the pneumonia before it progresses to lobar consolidation 90-95% caused by Streptococcus pneumoniae - the mucoid capsule of the bacteria protects them from phagocytosis and favors their dissemination; other bacteria which cause lobar pneumonia include Klebsiella pneumoniae, staphylococci, other streptococci, H. influenzae, Pseudomonas, Proteus

26. Lobar pneumonia: The entire left upper lobe is pale and firm (consolidated), reflecting the acute inflammatory exudate filling virtually all airspaces. There is focal abscess formation (box).

27. Empyema: Suppurative exudate covers the pleural surfaces. This exudate may organize and form a pleural rind and pleural adhesions that can impair lung expansion.

18

General progression of morphology in lobar pneumonias:


Gross: Red, swollen Red, firm, liver-like, often with overlying pleuritis Red hepatization Gray-brown Normal or variable fibrosis

Gray hepatization

Time
Micro: Congestion, edema, few neutrophils, bacteria Numerous PMN and RBC in alveoli, bacteria in PMN Degenerating PMN, fibrin

Legionnaire's Disease: pneumonia caused by Legionella pneumophila occurring primarily in smokers, elderly, patients with chronic lung diseases, and immunosuppressed patients; bacteria requires special media for culture, and a silver stain is needed for its detection in tissues Bacteria are resistant to chlorine, may contaminate cooling systems in buildings Aspiration Pneumonia Aspiration is common in people with: Alteration in mental status or level of consciousness producing a diminished or absent gag reflex or compromised epiglottic closure Neuromuscular disorders (myasthenia gravis, laryngeal palsy) Gastroesophageal reflux The location of the aspiration pneumonia depends on the patient's position and gravity: lying on back ---> apical segment of right lower lobe lying on side ---> posterior regions of upper lobe chronic ---> both lower lobes Causative agents are usually gram negative enterics and anaerobes, but Pseudomonas and S. aureus are important in hospitalized patients. In addition to the usual changes of acute pneumonia, other pathologic presentations can include diffuse alveolar damage/ARDS (with aspiration of gastric acid), foreign body giant cell reaction to food particles, and lipoid pneumonia (see below).

Lipoid pneumonia (exogenous): an oily substance (ex: mineral oil, used by some folks
as a laxative) is aspirated and phagocytosed by macrophages, whose cytoplasm then appears foamy; fibrosis can also develop over time

19

Endogenous lipoid pneumonia is histologically similar to exogenous lipoid pneumonia, but is usually secondary to bronchial obstruction (not aspiration)
28. Lipoid pneumonia: In exogenous lipoid pneumonia, aspirated oily substances (ex: mineral oil) are phagocytized by macrophages whose cytoplasm takes on a foamy appearance. Large empty spaces (*) represent collections of fat that did not survive the processing steps involved in making a glass slide. The fat triggers fibrosis. Endogenous lipoid pneumonia is usually caused by bronchial obstruction by a tumor or other process, and is histologically similar to exogenous lipoid pneumonia, except that the large fat collections are absent.

Bacterial causes of chronic pneumonias: Nocardia asteroides: aerobic gram + bacteria that grows in branched chains and causes opportunistic infections in immunocompromised people (defective T-cell mediated immune function due to HIV, prolonged steroid use, diabetes mellitus) o Causes respiratory disease that often resembles TB (chronic cough, fever, weight loss, pulmonary infiltrates), and also often infects the CNS o Stains with Fite acid fast stain (unlike Actinomyces) Actinomyces israelii: gram + anaerobic filamentous bacteria that are commonly found in the mouth but can cause aspiration-related pneumonia or abscess o Similar morphology to Nocardia sp., but form sulfur granules and do not stain with the Fite stain (unlike Nocardia)

29. Nocardia asteroides: The bacteria form chains and are acid fast-positive.

30. Actinomyces israelii: A sulfur granule formed by the bacteria is shown.

LUNG ABSCESS:
tissue

localized suppurative process with necrosis of lung

Most frequent in young adults Mixed infections are common, and 60% are caused by anaerobic organisms normally found in the oral cavity Predisposing factors

20

o Oropharyngeal surgeries, sinobronchial infections, CNS impairment aspiration of oropharyngeal contents o Preexisting pulmonary bacterial infection - pneumonia or bronchiectasis caused by a virulent organism necrosis of lung tissue o Neoplasm (present in 10-15% of abscesses) persistent airway infection o Septic embolism from systemic veins or the right side of the heart Symptoms: cough, fever, copious foul-smelling purulent sputum, chest pain CXR: cavitary mass with air-fluid level

Morphology: Gross: necrotic, often cavitary, masses measuring from millimeters to 6 cm., which may be filled with purulent material, or if a communication exists with an air passage, may contain air (drainage having occurred through the airway); when due to aspiration, more common on the right (main bronchus is more vertical) Microscopic: suppurative (neutrophilic) necrosis of lung parenchyma

32. Abscess: Neutrophils are mixed with necrotic material. The giant cells raise the possibility of aspirated material (though none is seen in this view). 31. Abscess: An abscess cavity (*) is formed by suppurative necrosis of lung tissue, in this case caused by a bacterial infection that was probably related to aspiration (note the location in the RLL). Next to the abscess, the lung parenchyma appears consolidated (firm and tan) due to pneumonia. Two small bronchi terminate in the abscess, passages through which foul smelling purulent material can be coughed up.

ATYPICAL PNEUMONIAS
More common causes of atypical pneumonia:
AGENT Mycoplasma pneumoniae Chlamydia spp. (pneumoniae, psittaci, trachomatis) Coxiella burnetti (Q fever) RNA viruses: Respiratory syncytial virus, Parainfluenza Human metapneumovirus Influenza DNA viruses: Adenovirus Herpes simplex virus Varicella-zoster virus Cytomegalovirus Children, military recruits Adults All Adults, esp. pregnant women All Nonimmunocompromised Immunocompromised Immunocompromised Both Immunocompromised Infants, children Adults Children and adults Adults Nonimmunocompromised Immunocompromised Nonimmunocompromised Nonimmunocompromised USUAL PATIENT GROUP Adults Adult Adults USUAL IMMUNE STATUS Nonimmunocompromised Nonimmunocompromised Nonimmunocompromised

Viral Pneumonias: General Comments


Most of the respiratory viruses (RSV, parainfluenza, influenza, adenovirus, rhinovirus) can produce clinical illnesses ranging from trivial upper respiratory tract infections to severe lower respiratory tract infections. Clinical severity is highly influenced by host factors, particularly immune status. Malnutrition, alcoholism, and debilitating illnesses also favor development of a more severe infection. Superimposed bacterial pneumonias are a major source of morbidity and mortality with some of the viral pneumonias, especially influenza A. Viral pneumonias can be diffuse and bilateral, patchy, or localized. Depending upon the virus, the primary site of infection can be the airways, the alveolar septa, or both. Common patterns of lung involvement by viruses are listed below. Identification of one of these patterns should prompt a search for cytologic evidence of a specific virus. Diffuse alveolar damage bilateral severe pneumonia pattern that is more common with immunocompromised patients

Interstitial pneumonia - interstitial inflammation Bronchitis/bronchiolitis/bronchopneumonia - inflammation in airways; acute bronchiolitis can lead to bronchiolitis obliterans (bronchiolar lumenal obliteration by fibrous tissue) Hemorrhagic parenchymal nodules - foci of hemorrhage, inflammation, necrosis

Respiratory syncytial virus (RSV)


Most common cause of serious lower respiratory tract infection in children < 1 yr.; occasional cause of significant pneumonias in immunocompromised adults Common cause of upper respiratory tract infections in children and less often in adults Characteristic cytologic findings: multinucleated giant cells (virus produces a fusion protein); eosinophilic cytoplasmic inclusions may be present

33. Respiratory syncytial virus (RSV) induced diffuse alveolar damage: Alveolar septa are widened and show prominent fibroblast proliferation. Multinucleated giant cells characteristic of RSV are present. The inset shows a giant cell with an eosinophilic intracytoplasmic inclusion.

Influenza A
Most common viral pneumonia in the nonimmunocompromised adult Although multiple viral subtypes exist at any one time, one usually predominates; subtypes are defined by hemagglutinin and neuraminidase antigen composition Causes "flu syndrome" - fever, myalgias, headaches, cough, usually in winter or spring Fatalities may be due to the viral pneumonia or to a bacterial superinfection Characteristic cytologic findings: none

Pandemics and epidemics occur with antigenic shift (recombination of RNA segments with those of an animal virus) and antigenic drift, respectively, that render existing antibodies to hemagglutinin and neuraminidase antigens less protective. Hemagglutinin (H) and neuraminidase (N) are antigens present in the viral envelope. Antibodies to H usually prevent future infection, and antibodies to N ameliorate future infection with the virus. With mutations in H and/or N, existing antibodies may no longer be effective, rendering the host again susceptible to infection with the new virus. Hemagglutinin mediates viral entry into the host cell, and neuraminidase may be important in releasing viruses from host cells.

34. Influenza A induced necrotizing tracheobronchitis and diffuse alveolar damage: The lungs are red and firm. The tracheal and bronchial mucosa is marked hyperemic due to necrosis of the bronchial epithelium.

Adenovirus
Usually causes mild upper respiratory tract infections, but can cause severe sporadic pneumonia esp. in immunocompromised patients Military recruits often acquire this infection Characteristic cytologic findings: "smudge cells" (large, basophilic, intranuclear inclusions make nuclear membranes appear smudgy)

35. Adenovirus induced necrotizing bronchiolitis and pneumonia: A bronchiole and surrounding alveoli are filled with acute inflammatory exudate. Architectural landmarks are obscured by necrosis.

36. Adenovirus induced necrotizing pneumonia: Several "smudge cells" (circled) are present. The nuclei of these cells are filled with basophilic intranuclear inclusions that obscure the nuclear envelope.

Herpes simplex virus (HSV)


Forms of infection: Ulcerative tracheobronchitis +/- necrotizing pneumonia: usually caused by aspirated oral secretions containing HSV type 1, in seriously ill patients Hemorrhagic parenchymal nodules: develops in profoundly immunocompromised patients with HSV viremia

Characteristic cytologic findings: "ground glass" nuclei, eosinophilic intranuclear inclusion surrounded by halo, multinucleated giant cells
epithelium has ulcerated (near blue dots). Cells in the sloughed epithelium have granular intranuclear inclusions (circled). A multinucleated giant cell (arrow), with intranuclear inclusions within each of its nuclei, is present. Several cells have "ground glass" nuclei.

37-38. Herpes simplex virus (HSV) induced ulcerative tracheobronchitis: The bronchial

39. Herpes simplex virus (HSV) pneumonia: Hemorrhagic parenchymal nodules occur as a consequence of viremia, usually in immunocompromised patients. The parenchyma shown here is necrotic, hemorrhagic, and inflamed. The inset shows nuclei with a "ground glass" appearance.

Varicella zoster virus


Primary varicella infection in children is rarely complicated by significant pulmonary involvement, but 15% of infected adults show radiologic evidence of pneumonia; this difference may result from the relatively large inoculum of virus that adults receive from infected children Pregnancy predisposes to development of pneumonia, mortality rate is 42% Hemorrhagic parenchymal nodule form of infection and cytologic findings are the same as HSV

Cytomegalovirus (CMV)
Almost exclusively in immunocompromised patients and neonates; AIDS patients and transplant patients are especially susceptible Characteristic cytologic findings: cellular and nuclear enlargement, intranuclear inclusions (central, large, often with halo) and intracytoplasmic inclusions (perinuclear basophilic specks)

40. Cytomegalovirus (CMV) pneumonia: Four cells, probably pulmonary macrophages, show cytologic alterations indicative of CMV infection (cellular and nuclear enlargement, basophilic intranuclear inclusion, and small intracytoplasmic inclusions).

MYCOPLASMA PNEUMONIA (Primary Atypical Pneumonia)


Common cause of community-acquired pneumonia, accounting for 18% of all hospital admissions for community-acquired pneumonia Considered to be a very small bacterium Generally produces a mild pneumonia with symptoms of cough, fever, malaise, and sometimes rash, arthritis, or hemolytic anemia, and the mortality rate is low Because the organisms adhere to respiratory epithelial cells and inhibit ciliary activity, the infection predisposes to bacterial superinfection Antibodies to M. pneumoniae (cold agglutinins - serum agglutinates group O red blood cells at a temperature between 0-5C) help to establish the diagnosis and are detectable in 50% of infected people

PNEUMOCYSTIS JIROVECI (FORMERLY CARINII) PNEUMONIA


Pneumocystis jiroveci is a ubiquitous opportunistic fungus that causes an atypical pneumonia in AIDS patients with CD4+ (helper) T cell counts below 200/mm3, children with malnutrition, and immunosuppressed patients Often the first diagnosed opportunistic infection in HIV-positive patients (the presenting feature in about 50% of cases), develops in 70-80% of AIDS patients, and is the leading cause of death in people with AIDS Most people become infected at some point, but do not develop disease unless they have one of the predisposing factors listed above Chest X ray: perihilar shadows Usually diagnosed in bronchoalveolar lavage fluid or lung biopsy, which shows foamy eosinophilic material in alveolar spaces, containing cyst forms that are visible with a silver methenamine stain o Diffuse alveolar damage or interstitial inflammation is usually present

41. Pneumocystis jiroveci pneumonia: Alveoli contain eosinophilic exudate that has a honeycomb appearance. The open spaces in the exudate are occupied by cysts. Cysts cannot be visualized with a hematoxylin-eosin stain.

42. Pneumocystis jiroveci pneumonia [Pap stain and silver methenamine stain (inset)]: This view came from a bronchoalveolar lavage sample, and shows the characteristic exudate associated with Pneumocystis jiroveci pneumonia. The exudate has a honeycomb appearance. Round cysts with a central dot, characteristic of Pneumocystis, are shown in the inset.

PULMONARY TUBERCULOSIS
Definition Infection caused by Mycobacterium tuberculosis hominis (MTB), a slender beaded rod which stains with the Ziehl-Neelsen (acid fast) stain and has a waxy coat comprised of multiple complex lipids and carbohydrates, making it impermeable to many common drugs; does not stain with a Gram stain Incidence and Location About one-third of people throughout the world are infected with MTB Tuberculous disease develops in ~ 10% of infected individuals Infection is most commonly diagnosed in the elderly and people with HIV Incidence is higher in foreign-born individuals than in individuals born in the USA Primary infection is more common in young adults in the U.S. Transmission Inhalation of organism-bearing particles from a patient with an airwaycommunicating lesion, via coughing, sneezing, or speaking Rare infection via an open skin wound or mucous membrane Clinical Features Typical symptoms include cough, fever, anorexia, fatigue, weight loss, chest pain, and night sweats; purulent sputum, hemoptysis, and clubbing may be present Some patients are asymptomatic Prognosis and Therapy Excellent prognosis for most patients, provided treatment guidelines are adhered to and infections are not due to multidrug-resistant strains Poorer prognosis with multidrug-resistant organisms Drug therapy decisions are made primarily based upon disease presentation and drug sensitivities Modified from Jagirdar J: Mycobacterial diseases. In Zander DS, Farver CF: Pulmonary Pathology, Elsevier, 2008.
43. Mycobacterium tuberculosis (acid fast stain): This Ziehl-Neelsen stain highlights numerous mycobacteria. The mycobacteria are long thin bacilli with a beaded appearance when viewed under high power. The clustering of some of these bacilli reflects the fact that they occupied and proliferated in the cytoplasm of a macrophage.

Pathogenesis: Type IV hypersensitivity reaction usually develops 2-4 weeks after primary infection and provides resistance to organisms but also causes tissue damage via granulomas. Travel plans: Adjacent lung Draining nodes Elsewhere Phagocytosis by macrophages, PMN

Inhalati on of lymph bacilli TB

Caseous necrosis Epithelioid macrophages destruction

Granuloma
Limits spread of infection Causes local tissue Indicates type IV hypersensitivity

T-cells are important in the immune response to mycobacteria: CD4+ T-cells secrete interferon-gamma which activates macrophages to kill intracellular organisms CD8+ T-cells kill infected macrophages and mycobacteria caseation relative acidity and low oxygen level are not favorable for mycobacteria Patients with reduced T cell numbers/functionality may not form granulomas, so organisms may not be as well-contained and may be more likely to spread Mycobacteria produce a variety of substances that assist in their survival, including: "Cord factor" - needed for in vitro growth of TB in serpentine cords; assoc. with virulence Sulfated glycolipids (sulfatides) - prevent fusion of phagosomes with lysosomes and promote intracellular survival of mycobacteria within macrophages

Forms of Disease
Primary tuberculosis New infection that is usually clinically silent and discovered incidentally via a +PPD or on chest x-ray The focus of primary infection is usually found in the lower part of an upper lobe or the upper part of a lower lobe, adjacent to the pleura (regions which receive the greatest flow of air) o Gross: 1.0 - 1.5 cm. gray-white focus of consolidation o Microscopic: neutrophils caseating granuloma with acid fast bacilli o Bacilli drain tracheobronchial lymph nodes develop granulomas Potential outcomes: o Localized infection (most common outcome): infected site fibrosis and calcification; the bacilli are thought to persist in a dormant but viable state for years (may indefinite PPD reactivity) o Progressive primary tuberculosis (more common with HIV): primary lung focus enlarges and spreads; lack of apical lesions distinguishes this form of disease from secondary TB

44. Caseating granuloma (tuberculosis): A well-circumscribed yellow nodule with a necrotic center lies in the lung tissue.

45. Caseating granuloma (tuberculosis): A granuloma is a collection of epithelioid histiocytes with multinucleated giant cells and a peripheral rim of lymphocytes. The center of this granuloma is necrotic, with a cheesy consistency that is referred to as caseous necrosis. Although caseating granulomas are characteristic of M. tuberculosis, this organism can also elicit non-necrotizing granulomas, as well as less organized macrophage infiltrates, if cellular immunity is impaired.

46. M. tuberculosis infection with hilar lymphadenopathy: A hilar lymph node demonstrates partial replacement by white necrotic material (*), which represents necrotizing granulomas caused by tuberculosis.

Secondary (postprimary or reactivation) tuberculosis Tuberculous infection in a previously sensitized individual, with bacilli derived from endogenous (more common) or exogenous (less common) sources 10% lifetime risk for individuals infected with TB More likely to develop when defenses are lowered Usual presentation: caseating granulomas in the apical segments of the upper lobe(s); the relatively higher p02 in this area is believed to favor bacterial proliferation here Potential outcomes: o Fibrocalcific ("arrested") tuberculosis: granulomas undergo fibrosis, calcification; although organisms can be hard to demonstrate histologically in these lesions, they can often be isolated by culture o Progressive pulmonary tuberculosis: infection spreads Cavitary fibrocaseous tuberculosis: caseating granuloma erodes into a bronchus and drains, forming a cavity increased oxygen tension fosters proliferation of large numbers of bacteria which can be coughed up and spread infection to others Tracheobronchial tuberculosis: the infected material gains access to the airways, usually from a cavity, and seeds the mucosal linings of the bronchi, trachea, and larynx Tuberculous pneumonia: consolidation of a lobe or portion of a lobe, usually in a patient with reduced host defenses or immunity Pleural involvement o Miliary tuberculosis: erosion of a granuloma into a lymphatic or blood vessel pulmonary or systemic dissemination numerous 1-3 mm. yellow-white lesions (granulomas) o Progressive isolated-organ tuberculosis: one organ develops progressive involvement, while other foci in other organs probably heal Most common: meninges, kidneys, bones, vertebra (Pott's disease) o Amyloidosis: rare

47. Postprimary tuberculosis: At the apex of the lung, there is a pleural plaque associated with a zone of consolidation in the underlying lung.

48. Cavitary tuberculosis: There is a large cavity in the inferior upper lobe. A cavity forms when a necrotizing granuloma(s) erodes through a bronchial wall, and the necrotic tissue is expectorated via a communicating bronchus. Cavities usually contain numerous mycobacteria. 49. Tuberculous pneumonia: This lobe is extensively consolidated, due to widespread infection of air spaces (bronchioles and alveoli) by M. tuberculosis.

50-51. Miliary tuberculosis: Numerous small (2 to 3 mm) foci of granulomatous inflammation dot the lung parenchyma. Each of these foci contains tuberculous bacilli. Miliary dissemination of tuberculosis may occur within the lungs and systemically. Miliary dissemination usually occurs due to spread of the organisms through the lymphatic and/or vascular systems. 52. Tuberculous empyema: The infected pleural space is markedly expanded and filled with white material representing caseating granulomas (*) containing tuberculous bacilli, and red-brown hemorrhagic material. Additionally, infected hilar lymph nodes are present (white areas).

Tuberculosis and HIV TB is frequently an early indication of HIV infection Higher rate of development of active disease in infected patients (8%/year vs. 10% lifetime risk for HIV - negative people) Much higher rate of extrapulmonary (pleural, lymphadenitis, meningitis, miliary spread) TB than HIV - negative people Patients are frequently anergic, interfering with detection of infection by PPD Lymphokine secretion by infected CD4+ T cells may be inadequate to activate macrophages to kill mycobacteria, and/or HIV-infected and mycobacteriainfected macrophages may fail to respond to the lymphokines; increased CD8+ T cells may also result in increased macrophage destruction in the lesions

ATYPICAL MYCOBACTERIA
Non-tuberculous (atypical) mycobacteria are widely distributed in the environment (soil, water, plants, and animal excretions) Cause opportunistic pulmonary infections in patients with AIDS and in middle-aged or elderly people with chronic lung diseases; M. avium complex is the most common pathogen, but many other species can also cause disease Pathology: similar to tuberculosis Difficult to treat due to drug resistance and patient debilitation

FUNGAL INFECTIONS OF THE LUNG


Some fungal species are pathogenic (capable of infecting normal persons) while others are opportunistic and particularly problematic in patients receiving immunosuppressive drugs or having other diseases that compromise immunity. For many fungi, a spectrum of colonization and disease exists, with host defenses playing an important role in the type of manifestations a patient will develop. The nature and intensity of the host response is influenced by immune status, and ranges from acute pyogenic to chronic granulomatous infection. Some species are found in characteristic geographic regions (endemic) Diagnosis: tentative microscopic species identification can by made based on fungal morphology in tissues, using special stains such as methenamine silver (MSS) or PAS, but cultures are important for confirmation; however, commensal or free-living fungi can also be isolated by culture, so the culture results must be correlated with the patient's clinical condition to assess their relevance

Common fungal infections in the United States


Species Histoplasma capsulatum Cryptococcus neoformans Blastomyces dermatitidis Coccidiodes immitis Aspergillus fumigatus, niger, flavus Zygomycetes (Mucor, Rhizopus, Absidia) Forms small yeast with single narrowbased bud encapsulated yeasts of variable sizes with single narrow-based bud yeast with single broad-based bud spherule with endospores branching septate hyphae; occasionally conidia broad branching nonseptate hyphae; rare sporangia Geographic Distribution Ohio-Mississippi River region world-wide Mississippi-Ohio River basins, mid-Atlantic states Southwest, West (esp. San Joaquin Valley, CA), Mexico, S. America world-wide world-wide

Histoplasmosis (Histoplasma capsulatum)


Infection occurs by inhalation of fungal spores in dust contaminated with bird or bat droppings intracellular parasites of macrophages necrotizing granulomas containing small yeasts fibrosis and/or calcification with therapy or resolution of infection Forms of infection are similar to tuberculosis and depend on host defenses and immune status: Latent asymptomatic infection: one or more fibrotic and/or calcified foci of infection in the lungs or lymph nodes; common Self-limited primary pulmonary involvement: mild febrile respiratory infection with hilar adenopathy and pulmonary parenchymal infiltrate(s); common Chronic progressive pulmonary disease: apical infiltrates that may enlarge and can cavitate, disseminate, or spontaneously improve; resembles secondary tuberculosis Rapid widely disseminated systemic histoplasmosis: usually an acute progressive infection in the very old, very young, or people with reduced cellular immunity such as AIDS patients Localized extrapulmonary lesions

54. Histoplasmosis: Multiple necrotizing granulomas are scattered throughout this section of lung.

53. Histoplasmosis: Multiple granulomas (arrows) are present in multiple lobes.

55. Histoplasma capsulatum (silver methenamine stain): The tissue form of Histoplasma is a small yeast. The yeast reproduces by budding, which can be seen in this view. Each of the clusters of yeasts represents the contents of a single macrophage, which has since died.

56. Histoplasmosis: The patient from whom this biopsy was taken had impaired cellular immunity, and could not produce wellformed granulomas. Instead, his response to the fungal infection was diffuse macrophage infiltration of the alveolar spaces, with smaller numbers of other inflammatory cells. The infection could not be contained, and spread between alveolar spaces and through bronchioles, like a bacterial pneumonia.

Cryptococcosis (Cryptococcus neoformans)


Inhalation of cryptococci in bird droppings, esp. pigeons, leads to: Limited granulomatous infection in healthy people Opportunistic infection in patients with leukemia, lymphoma, or especially AIDS; can become progressive in the lungs and disseminate systemically; meningitis is common, and the India Ink test may reveal the fungi in the cerebrospinal fluid; granulomas are usually less well-formed or absent

57. Cryptococcosis (hematoxylin-eosin with mucicarmine stain inset): Cryptococcus neoformans usually has a mucoid capsule. Dissolution of the capsule during processing produces an artifactual halo (arrows) in tissue sections. Cryptococcus is the only pathogenic yeast stainable in tissue with a mucicarmine stain (red capsule-see inset).

Mucormycosis (Zygomycosis, Phycomycosis)


Caused by the "breadmold fungi" Mucor, Rhizopus, Absidia, and Cunninghamella Opportunistic infection occurring in patients with diabetic ketoacidosis, advanced malignancy, leukemia, lymphoma, immunodeficiency, or inpatients receiving broad spectrum antibiotic therapy, steroids, or cytotoxic chemotherapy Forms of infection: inhalation of spores may produce rhinocerebral disease (nasal, sinuses, orbits, brain) especially in patients with diabetic ketoacidosis, or pulmonary disease; ingestion may result in gastrointestinal disease Surgical debridement of infected tissue and antifungal therapy can occasionally produce a cure, but generally the prognosis is poor Broad irregularly (often 90 degree) branching nonseptate fungal hyphae invade the parenchyma and arterial walls (angioinvasion) -----> thrombosis and infarction, hematogenous dissemination

58. Angioinvasive mucormycosis: The entire left lung is infarcted, due to extensive vascular invasion by mucormycosis and subsequent pulmonary artery thrombosis. Vascular invasion, thrombosis, and subsequent infarction of the infected organ are characteristic manifestations of mucormycosis. Dissemination to other sites also occurs via vascular channels. This patient was markedly immunocompromised.

59-60. Angioinvasive mucormycosis (hematoxylin eosin above, MSS-below): The top image shows a mediumsized blood vessel in the lung whose lumen is filled with numerous intertwined fungal hyphae (*) admixed with red cells. The MSS stain for fungus (lower image) highlights the hyphae (black) that fill the vessel lumen and infiltrate the vessel wall. The hyphae are broad, irregularly branching, and nonseptate, features of zygomycetes, the family of fungi which includes Mucor, Rhizopus, and Absidia.

Aspergillosis (Aspergillus fumigatus, niger, and other species)


Widely distributed in nature, found in dust raised during construction, and frequently contaminate hospitals Fungal morphology: thin septate hyphae with parallel walls and branching at approx. 45 degree angles Usual Patients Asthmatics Pathology Superficial colonization of the bronchial mucosa -----> inflammation and scarring of the airway, worsening of symptoms Cavity or ectatic bronchus becomes filled with proliferating masses of fungal hyphae; tissue invasion is absent or minimal Airway invasion by fungi, usually relatively limited, often chronic Parenchymal invasion by fungi with slow growth, often contained in granulomas Necrotizing pneumonia with tissue and vascular invasion by fungus, thrombosis and infarction Systemic dissemination via hematogenous spread, with parenchymal invasion, vascular invasion, thrombosis and infarction

Forms of Disease Allergic bronchopulmonary aspergillosis Aspergilloma ("fungus ball")

Have pre-existing cavity or bronchiectasis Slightly compromised Slightly compromised

Airway-invasive aspergillosis Chronic necrotizing aspergillosis Invasive aspergillosis

Immunocompromised

Disseminated aspergillosis

Immunocompromised

61. Aspergillus infection (silver methenamine stain): Aspergillus species demonstrate thin septate hyphae with approximate 45 angle branching.

62-63. Aspergilloma (fungus ball): A large aggregate of aspergillus hyphae fills a preexisting cavity in the lung apex (arrow and *).

64. Invasive aspergillosis: This lung from a severely immunocompromised patient has numerous round colonies of fungus.

DIFFUSE INTERSTITIAL DISEASES OF THE LUNG


Diseases with recognized causes
1. Occupational and environmental substances Inorganic dust diseases (asbestosis, silicosis, coal workers pneumoconiosis, etc) Numerous antigens (hypersensitivity pneumonitis) Toxic inhalants (oxygen toxicity, sulfur dioxide, mustard gas, etc) 2. Drugs, esp. chemotherapeutic agents 3. Radiation pneumonitis 4. Infections: viral, mycoplasmal, Pneumocystis, occasionally bacterial, fungal, viral

Diseases with undetermined causes


1. Idiopathic interstitial pneumonias Usual interstitial pneumonia Desquamative interstitial pneumonia Acute interstitial pneumonia Nonspecific interstitial pneumonia Cryptogenic organizing pneumonia Respiratory bronchiolitis interstitial lung disease Lymphoid interstitial pneumonia 2. Sarcoidosis 3. Langerhans cell histiocytosis (Histiocytosis X) 4. Lymphangiomyomatosis 5. Alveolar proteinosis 6. Idiopathic hemosiderosis 7. Eosinophilic pneumonia, acute and chronic 8. Goodpastures syndrome

Common characteristics of diffuse interstitial lung diseases


Diffuse, often bilateral, involvement of the lung with primary effects on the interstitium in the alveolar walls Symptoms - dyspnea, tachypnea, and ultimately hypoxemia and cyanosis CXR - diffuse bilateral "ground glass" or interstitial nodular and irregular shadows Restrictive functional alterations in pulmonary function: decreased oxygendiffusing capacity, lung volumes, and compliance

Some of the diseases may lead to "honeycomb lung" (endstage replacement of alveoli by cystic spaces separated by thick fibrous bands with inflammatory cells) and right-sided heart failure secondary to pulmonary hypertension
65. Honeycomb lung: The air spaces are enlarged and separated by fibrous bands. This represents an end stage of several interstitial lung diseases.

In most of these diseases, lung injury is believed to result from an alveolitis, an accumulation of inflammatory and immune effector cells in alveoli, with release of their secreted products, stimulated by a known or unknown trigger. Neutrophils often contribute to the injury by release of oxygen-derived free radicals and toxic enzymes. Macrophages release similar substances as well as fibroblast growth factors that promote remodeling and scarring. Lymphocytes can cause injury to endothelial cells and epithelial cells and secrete substances that attract and activate macrophages and neutrophils, and cause granuloma formation in some diseases.

SARCOIDOSIS
A systemic granulomatous disease of unknown etiology, which commonly (in over 90% of cases) involves the lung and/or hilar or mediastinal lymph nodes Common, F > M, especially prevalent in the southeastern US o Association with HLA-A1 and B8 o Increased frequency in African-Americans than American Caucasians Causal agent is unknown but many have been suggested (mycobacteria, pine pollen, viruses); analysis for mycobacterial DNA has produced conflicting results Immunologic abnormalities suggest a cell-mediated response to an undetermined antigen: o Increased CD4+ T-cells in the lungs, with oligoclonal expansion of T-cell subsets o Increased Th1 cytokines (IL-1 and IFN-), leading to T-cell proliferation and macrophage activation in the lungs o Increases in other cytokines (IL-8, TNF, macrophage inflammatory protein 1, involved in the formation of granulomas o Cutaneous anergy to Candida or PPD o Polyclonal hypergammaglobulinemia, reflecting T-cell dysregulation Labs - hypercalcemia, increased angiotensin converting enzyme Course - variable; may be chronic, waxing/waning or spontaneously remitting, but usually corticosteroids produce improvement Morphology: Gross: Lungs - may have small nodules or no visible abnormalities Lymph nodes - hilar and mediastinal - enlarged ("potato nodes") Microscopic: small nonnecrotizing granulomas with perigranulomatous and interstitial fibrosis; may have asteroid or Schaumann's bodies (not specific for sarcoidosis) Since mycobacterial and fungal infections, and berylliosis (a pneumoconiosis caused by beryllium exposure) can produce similar granulomas, the diagnosis of sarcoidosis is one of exclusion (cultures and special stains reveal no organisms, no history of beryllium exposure)

66. Sarcoidosis: Compact non-necrotizing granulomas are the hallmark of sarcoidosis. Mycobacterial and fungal infections must be ruled out before making the diagnosis of sarcoidosis (diagnosis of exclusion).

IDIOPATHIC PULMONARY FIBROSIS (IPF) (synonyms: usual interstitial


pneumonitis, cryptogenic fibrosing alveolitis, etc.) Most patients are between 30 and 70 years old, M > F, and present with an insidious onset of dyspnea; course is usually chronic and progressive with survival times after diagnosis usually less than 5 years No effective therapy Etiology is unknown Current theory about pathogenesis is that repeated cycles of acute lung injury may lead to disadvantageous wound healing responses and scarring over time Immunologic mechanisms may have a role in pathogenesis o Th2 responses are thought to be involved o Activated macrophages attract neutrophils, leading to release of damaging proteases and oxidants o Activated macrophages and pneumocytes secrete fibrogenic and chemotactic cytokines, leading to fibroblast proliferation and collagen deposition o Histologically identical lung disease can develop in patients with rheumatoid arthritis and other collagen vascular disease o Immune complexes are increased in some patients Morphology: marked variation in the nature and severity of fibrosis and inflammation in adjacent areas of lung, which probably reflects the repetitive occurrence of acute lung injury over time (temporal heterogeneity of histologic changes), leading to airspace remodeling

Areas of relatively normal lung are found adjacent to abnormal areas, which have interstitial chronic inflammatory infiltrates (mostly lymphocytes with occasional plasma cells and other inflammatory cells), interstitial fibroblast proliferation (fibroblast foci) and collagen deposition, hyperplastic type II pneumocytes, bronchiolar metaplasia Honeycomb lung can develop with time

Destruction of the pulmonary capillary bed and hypoxemia pulmonary hypertension Similar pathologic findings may be associated with many other entities (collagen vascular diseases, oxygen toxicity pneumonitis, irradiation and chemotherapy-induced pneumonitis, pneumoconioses . . .), etiologies which must be excluded before making this diagnosis.

67. Usual interstitial pneumonia (idiopathic pulmonary fibrosis): The lung shows interstitial fibrosis and lymphocytes, as well as a fibroblast focus (arrow).

68. Usual interstitial pneumonia (idiopathic pulmonary fibrosis): There is subpleural interstitial fibrosis with honeycomb change (airspace remodeling).

COLLAGEN VASCULAR DISEASES


Lung and pleural involvement is common in patients with collagen vascular diseases. The spectrum of involvement includes pleuritis, interstitial pneumonia, vasculitis, and airway-centered lesions. In rheumatoid arthritis (forms of involvement below), abnormal pulmonary function tests are found in 30-40% of patients.
Pleural lesions Parenchymal lesions Chronic pleuritis with or without effusion Rheumatoid nodules Usual interstitial pneumonia Non-specific interstitial pneumonia Bronchiolitis Rheumatoid nodules Vasculitis Pulmonary hypertension Amyloidosis

CRYPTOGENIC ORGANIZING PNEUMONIA (formerly known as bronchiolitis obliterans organizing pneumonia


Clinical: cough, dyspnea, often a recent history of respiratory tract infection symptoms (occult viral infection is suspected as an etiology, but has not been proven) CXR: patchy consolidation or interstitial infiltrates Morphology: young connective tissue (fibroblasts with mucopolysaccharides and usually little collagen) in bronchioles, interstitium, and alveolar spaces Usually improves quickly with corticosteroids Similar histologic reactions can be associated with bacterial and viral infections, aspiration, collagen vascular diseases, drug reactions, inhaled toxins, etc., which must be excluded before diagnosing cryptogenic organizing pneumonia
69. Organizing pneumonia: The air spaces are filled with young connective tissue.

HYPERSENSITIVITY PNEUMONITIS (EXTRINSIC ALLERGIC ALVEOLITIS)


An immunologically-mediated disease that develops when a patient with abnormal sensitivity to an antigen is exposed to the inhaled antigen (usually an organic or occupational antigen, often spores of bacteria, fungi, animal proteins, etc) Acute (fever, cough, dyspnea 4-6 hrs after exposure) and chronic (slowly progressive dyspnea, cyanosis) presentations o Many patients have antibodies to the triggering agent in their serum, and this can be helpful in pinpointing the cause Treatment: stop exposure to the antigen, corticosteroids are often helpful Pathogenesis: believed to represent a type III hypersensitivity reaction (early) type IV hypersensitivity reaction to produce granulomas Morphology (chronic form): triad of a) interstitial pneumonitis (lymphocytes, plasma cells, macrophages) +/-- fibrosis, b) chronic or obliterative bronchiolitis, and c) granulomas or collections of giant cells

DISEASE Farmers lung Ventilation pneumonitis Bagassosis Bird fanciers, breeders or handlers lung Tobacco growers lung Coptic lung Thatched roof lung

ANTIGEN Bacteria - Micropolyspora Bacteria - actinomycete Bacteria - actinomycete Avian droppings, feathers, serum, etc

SOURCE Moldy hay, grain Contaminated air conditioner systems Moldy sugarcane Parakeets, pigeons, chickens, turkeys, etc Tobacco plants Cloth wrappings of mummies Dead grasses and leaves

PNEUMOCONIOSES: disorders caused by the inhalation of any dust or other aerosol


General characteristics Usually slow and insidious in onset, appearing after decades of exposure, and compatible with long life spans Although safety regulations have reduced the frequency of most pneumoconioses, the long time interval before they become clinically apparent means that they currently remain sources of morbidity and mortality Often occupational diseases For most substances capable of producing a pneumoconiosis, there is a spectrum of severity ranging from clinically insignificant depositions of material with little or no scarring to progressive, disabling and sometimes fatal scarring [progressive massive fibrosis (PMF)] associated with larger depositions of the offending agent o Symptoms of dyspnea and cough are usual with more fibrosis o PMF can lead to pulmonary hypertension and cor pulmonale Factors influencing the consequences of dust inhalation Amount of dust retained in the lung and airways: determined by the concentration of the dust in the air, duration of exposure, and effectiveness of clearance mechanisms (diminished by smoking) Size, shape and buoyancy of the particles: - particles 1-5u diam. tend to settle in small airways and alveoli - buoyant particles (ex: asbestos fibers) tend to stay in the center of a moving air stream, avoid clearance in the upper airways, and impact in the smaller airways Particles' solubility and physicochemical reactivity: - generally, smaller particles reach toxic levels more rapidly than larger particles; larger particles may persist and cause fibrosis - some particles (ex: silica) can directly trigger macrophages to release inflammatory mediators and fibrogenic substances Possible effects of other irritants (ex: carcinogens in cigarette smoke adsorbed on the surfaces of asbestos particles) may contribute to the synergy between smoking and heavy asbestos exposure in causing bronchogenic carcinoma

Pathogenesis: particles not removed by mucociliary clearance deposit in airways and are phagocytosed by macrophages -----> macrophages release toxic substances,

inflammatory mediators, and substances promoting fibroblast proliferation and collagen synthesis PNEUMOCONIOSES
AGENT Mineral Dusts Coal dust Silica Asbestos Beryllium Iron oxide Barium sulfate Tin oxide Organic Dusts that Induce Hypersensitivity Pneumonia Organic Dusts that Induce Asthma Cotton, flax, hemp Red cedar dust Chemical Fumes and Vapors Nitrous oxide, sulfur dioxide, ammonia, benzene, cyanate insecticides, numerous others macules, progressive massive fibrosis, Caplan's syndrome silicosis, Caplan's syndrome asbestosis, pleural plaques, Caplan's syndrome, mesothelioma, lung cancer acute and chronic berylliosis, cancer siderosis baritosis stannosis see section on hypersensitivity pneumonia byssinosis asthma bronchitis, asthma, pulmonary edema, respiratory distress syndrome, mucosal injury, fulminant poisoning textile manufacturing lumbering, carpentry occupational and accidental exposure coal mining sandblasting, mining, stone cutting, etc mining, milling, fabrication, installation and removal of insulation nuclear and aerospace industries welding mining mining DISEASE EXPOSURE

ASBESTOS: a generic term for a variety of naturally occurring fibrous silicates found
in the environment (air and water), sharing the properties of high tensile strength, high heat resistance, and resistance to chemical attack; everyones lungs contain some fibers Heaviest exposures occur in industries dealing with the mining, fabrication, and installation of asbestos and asbestos-containing products (insulation, flooring, ceiling, roofing, sewer and water conduits, brake linings, clutch casings)

Types of asbestos fibers: Serpentine (curly and flexible) fibers: tend to impact in the upper tracheobronchial tree removed by the mucociliary apparatus; chrysotile is often used commercially and is a major air pollutant Amphibole (brittle straight) fibers: may align with moving air and travel deeper into the lungs; crocidolite, amosite, and tremolite are used commercially ASBESTOS-RELATED PROBLEMS a. Asbestosis = diffuse interstitial fibrosis + asbestos bodies in areas of fibrosis Asbestos bodies: asbestos fibers phagocytized by macrophages and coated with iron-containing proteinaceous material; beaded, sometimes dumbbellshaped; a type of ferruginous body (particle with iron-protein coat) Most fibers remain uncoated (not usually visible by light microscopy)

70. Asbestos bodies: Asbestos bodies consist of asbestos fibers coated with hemosiderin-containing proteinaceous material.

71. Asbestosis: There is interstitial fibrosis and asbestos bodies.

b. Pleural reactions: fibrocalcific parietal pleural plaques - most common sign of asbestos exposure; visceral pleural fibrosis; effusions c. Cancers: also see sections on "Tumors" and "Pleural Diseases"

Bronchogenic carcinoma, especially adenocarcinoma, is the most common malignancy associated with asbestos exposure; 30-40% of people with asbestosis develop carcinoma. Relative risks: nonexposed nonsmokers asbestos-exposed nonsmokers nonexposed smokers asbestos-exposed smokers 1 5 11 55

Malignant mesotheliomas are rare tumors linked to amphibole exposure (>1000X increase in relative risk); families of heavily exposed workers also have an increased incidence. Also, laryngeal and other extrapulmonary neoplasms have been linked to asbestos exposure.

SILICOSIS: lung disease caused by inhalation of crystalline silica (SiO2)


Most prevalent chronic occupational disease in the world Exposures: mining, sandblasting, quarrying, stone cutting, etc.

Pathogenesis: silica is ingested by macrophages, activates them, and causes release of IL-1, TNF, fibronectin, oxygen-derived free radicals, fibrogenic cytokines, and other mediators; crystalline forms, including quartz, are more fibrogenic than amorphous forms Tuberculosis occurs with increased frequency (10 - 30 X increased incidence) in patients with silicosis International Agency for Research on Cancer stated that crystalline silica from occupational sources is carcinogenic in humans, but this is still a controversial subject Morphology: Gross: tiny collagenous nodules esp. in the upper zones -----> enlarge, coalesce -----> larger scars -----> honeycombing or massive fibrosis; draining lymph nodes also show collagenous nodules Microscopic: hyalinized collagenous nodules with concentric laminations separated by cleftlike spaces containing needle-shaped crystals of silicates (birefringent with polarization) and silica (poorly polarizable)

72. Silicosis: Younger, more cellular, and older, more fibrotic, silicotic nodules lie side by side.

COAL WORKERS PNEUMOCONIOSIS (CWP): lung disease caused by


inhalation of coal dust Anthracosis, simple CWP, and complicated CWP (PMF) are three sections of a continuum of severity of carbon dust accumulation and fibrosis in the lung. Fewer than 10% of people with simple CWP progress to PMF. Anthracosis: small clinically insignificant accumulations of carbon dust found in most of us, particularly in city dwellers and smokers Simple CWP: aggregates of coal dust - containing macrophages (macules), some with associated fibrosis (nodules), usually little respiratory impairment Few people

Many years of coal mining (usually > 30 years)

Complicated CWP: heavy accumulations of coal dust in the lungs with fibrous scars 2-10 cm.; clinically disabling dyspnea, chronic cough, blackened sputum

73. Coal workers pneumoconiosis: There is marked carbon deposition with scarring.

OBSTRUCTIVE PULMONARY DISEASES


Obstructive lung diseases include emphysema, chronic bronchitis, asthma, bronchiectasis, and small airway diseases (bronchiolitis), which share the symptom of dyspnea and the presence of chronic or recurrent obstruction to airflow in the lung. Chronic obstructive pulmonary disease (COPD) is a commonly used term that refers to emphysema and chronic bronchitis, two disorders which often coexist since they share the common etiologic factor of tobacco smoking. COPD is the fourth leading cause of morbidity and mortality in the United States.

EMPHYSEMA
Definition Disorder characterized by abnormal permanent enlargement of air spaces distal to the terminal bronchiole, with destruction of their walls and without obvious fibrosis Incidence Estimated to affect 5-8% of the U.S. population Primarily affects smokers and individuals with A1AT deficiency Morbidity and Mortality Can lead to pulmonary hypertension and cor pulmonale especially if concurrent chronic bronchitis exists Can cause death from respiratory failure or rupture of bullae with tension pneumothorax or hemorrhage Gender and Age Distribution Male predominance (male-to-female ratio 2:1) Symptoms in smokers usually develop after age 50, but can occur earlier in A1AT deficiency, especially with concurrent smoking Clinical Features Dyspnea is characteristic If chronic bronchitis coexists, individuals will have a productive cough Physical examination: barrel chest, decreased breath sounds, prolonged inspiration Spirometry: decreased forced expiratory volume in 1 second (FEV1) With advancing disease, hypoxemia develops Radiologic Features Hyperinflated lungs, tapering vascular shadows with hyperlucency, pushed

down diaphragms, bullae Prognosis and Therapy Damage is irreversible Smoking cessation is central to treatment plan, to avoid progression A1AT deficiency is treated with A1AT replacement therapy Surgical therapies include bullectomy, lung volume reduction therapy, and lung transplantation Pathologic Features Gross Findings o Enlarged and hyperinflated lungs o Bullae (distended air spaces > 1 cm diameter) may be present, especially in the apical regions in smokers, and may rupture and cause pneumothorax o Several types of emphysema, which can occur in combinations and with varying severities: Centriacinar (centrilobular): enlarged air spaces in centers of lobules (primarily at the level of the respiratory bronchiole), often with blackened walls, surrounded by unaffected parenchyma; upper lobes affected more than lower lobes Tobacco products are the usual cause Panacinar (panlobular): air space enlargement throughout the lobule; in A1AT deficiency, lower lobes are affected more than upper lobes Paraseptal: enlarged air spaces along interlobular septae and pleura Irregular: Localized air space enlargement adjacent to a focal lesion, usually a scar Microscopic Findings o Alveolar wall destruction with free-floating portions of septa that appear disconnected from other parenchymal structures, leading to air space enlargement Modified from Green L: Emphysema and diseases of large airways. In Zander DS, Farver CF: Pulmonary Pathology, Elsevier, 2008.

Centriacinar AD RB TB RB AD End-stage centriacinar or panacinar RB AD

Paraseptal

75. Centriacinar emphysema: This more severe case shows more extensive air space enlargement primarily in the centers of lobules, associated with deposits of carbon particles. 74. Centriacinar emphysema: Air spaces are enlarged, primarily in the center of the upper lobe. The enlarged air spaces are associated with deposits of carbon particles, giving the parenchyma a darkened appearance. Particles in tobacco smoke deposit primarily in respiratory bronchioles, which lie at the centers of lobules/acini. This initiates the development of emphysema adjacent to respiratory bronchioles. 76. Centriacinar emphysema: Air spaces around the respiratory bronchiole are enlarged, due to destruction of alveolar septa. Note the presence of black carbon deposits near the enlarged air spaces.

77. Panacinar emphysema: The parenchyma has been extensively destroyed, with enlargement of airspaces from the level of the respiratory bronchiole distally (to involve the entire acinus).

78. Panacinar emphysema: The parenchyma has been extensively destroyed, with enlargement of airspacesthroughout the acinus. Alveolar septa are fragmented and reduced in number.

79. Bullous emphysema: Bullae are air-filled spaces measuring greater than 1 cm. in diam. They may be associated with any type of emphysema. Complications arising from bullae include pneumothorax (from rupture of a bulla), compression of adjacent lung, and infection. The bulla (*) shown measures ~ 12 cm.

Pathogenesis: Protease-antiprotease theory - imbalance between proteases (mainly elastase) and antiproteases (mainly A1AT) in the lung leads to alveolar wall destruction Alpha-1-antitrypsin (A1AT) is the major antielastase in serum and interstitial tissue. Neutrophils, and to a lesser extent macrophages, mast cells, and bacteria, are the main sources of elastase. When elastase is given intratracheally

to experimental animals, emphysema results. Stimuli that increase the number of neutrophils and macrophages in the lung, or promote release of elastase, will increase tissue destruction, particularly if antiprotease activity is low (A1AT deficiency). A1AT deficiency: A1AT is transmitted codominantly and is specified by the proteinase inhibitor (Pi) locus on chromosome 14. About 70 A1AT phenotypes exist. The normal phenotype (PiMM) occurs in 90% of the population. Several phenotypes are associated with A1AT deficiency. PiZZ is the most common of these, and leads to emphysema in over 80% of people with this phenotype. Smoking accelerates development of emphysema in people with A1AT deficiency. The emphysema is believed to result from chronic low grade proteolysis due to neutrophil passage through the lungs (greatest in lower lobes, which show the most severe involvement). Cigarette smoking and centriacinar emphysema: Smokers have both increased pulmonary elastase activity and decreased antielastase activity. They also develop an imbalance between oxidants (increased) and antioxidants (decreased) in the lung. CD8+ T-cells may also contribute to emphysema by causing apoptosis of alveolar epithelial cells or through recruitment of macrophages. Smoke particles impact in small bronchi and bronchioles, with the following consequences which lead to centriacinar emphysema: neutrophils and macrophages accumulate in alveoli elastase and other protease release from neutrophils is stimulated macrophage elastase activity (not inhibited by A1AT) is enhanced oxidants (reactive oxygen species) in smoke and released by neutrophils produce oxidative lung damage and inhibit A1AT

CHRONIC BRONCHITIS
Definition Clinically defined by a chronic productive or mucus-producing cough on most days of the month for at least 3 months in at least 2 consecutive years, with no other identifiable cause Incidence Affects 4-5% of the U.S. population Morbidity and Mortality A component of COPD, which is the 4th leading cause of death in the United States and has a mortality rate of 50% at 10 years after diagnosis Can lead to pulmonary hypertension and cor pulmonale Can cause death from respiratory failure due to severe disease, often with superimposed respiratory tract infection Age Distribution Most affected individuals are more than 45 years of age Clinical Features Associated with smoking (more than 90%); exposure to dusts, fumes, or toxins; airway infections; allergic airway injury Symptoms include cough with excessive sputum production, dyspnea Superimposed respiratory tract infections are common, and can be associated with worsening respiratory symptoms, fever, chills Spirometry: decreased forced expiratory volume in 1 second (FEV1) With advancing disease, hypoxemia develops Radiologic Features Nonspecific but may show bronchial wall thickening Therapy Geared toward reducing bronchial irritation and treating infections: smoking cessation, antibiotics for infections, bronchodilators, supplemental oxygen Pathologic Features Gross Findings Airways are filled with abundant mucus or mucopurulent secretions Bronchial walls often appear thickened, and the mucosal surfaces hyperemic Microscopic Findings Abundant mucus in bronchial and bronchiolar lumens, with variable acute inflammation depending upon the presence or absence of infection Enlargement of the mucous glands in the bronchial wall Increased bronchial goblet cells, goblet cell metaplasia of bronchioles

Chronic inflammatory infiltrates in airway mucosa and submucosa Increased bronchial-associated lymphoid tissue (BALT) in small airways Submucosal and adventitial fibrosis in bronchi and bronchioles Squamous metaplasia and reserve cell hyperplasia Thickening of the bronchial basement membrane Modified from Green L: Emphysema and diseases of large airways. In Zander DS, Farver CF: Pulmonary Pathology, Elsevier, 2008. Pathogenesis: Inhaled substances (especially cigarette smoke but can also be allergens in people with asthma) cause chronic irritation of the tracheobronchial system. Infections help maintain and acutely exacerbate the inflammation and injury. Cigarette smoking predisposes to infections by impairing ciliary function, damaging respiratory epithelium, and interfering with the ability of leukocytes to clear bacteria. Smoking, allergen(s), air pollutant(s) or other chronic irritant(s) respiratory bronchiolitis (small airways disease) with intraluminal pigmented macrophages, bronchiolar chronic inflammation and fibrosis increased size of submucosal glands and increased goblet cells hypersecretion of mucus in large airways sputum overproduction, obstruction of small airways variable epithelial changes (metaplasia, dysplasia)

80-82. Chronic bronchitis: Although chronic bronchitis is a clinically defined disorder, pathologic findings associated with it include an increase in the size of the bronchial submucosal glands to occupy greater than 40% of the thickness of the bronchial submucosa, increased mucus or mucopurulent secretions in bronchi and bronchioles, increased goblet cells, thickened basement membrane, chronic inflammation, and epithelial changes (metaplasia, dysplasia).

ASTHMA
Definition National Asthma Education and Prevention Program Expert Panel (1997): Asthma is a chronic inflammatory disorder of the airways in which many cells and cellular elements play a role, in particular, mast cells, eosinophils, T lymphocytes, neutrophils, and epithelial cells. In susceptible individuals, this inflammation causes recurrent episodes of wheezing, breathlessness, chest tightness, and cough, particularly at night and in the early morning. These episodes are usually associated with widespread but variable airflow obstruction that is often reversible either spontaneously or with treatment. The inflammation also causes an associated increase in the existing bronchial hyperresponsiveness to a variety of stimuli. Incidence Affects 5-10% of the U.S. population, and is increasing Morbidity and Mortality Responsible for more than 11 million hospital visits per year Accounts for > 5,000 deaths annually, and may contribute to more Status asthmaticus is the most severe and potentially fatal form; attacks can last for days Gender and Age Distribution Male predominance (M:F ratio 2:1) until puberty (M:F ratio 1:1) Higher frequency in children, with 2/3 of individuals diagnosed before age 18 Clinical Features Symptoms: wheezing, breathlessness, cough, chest tightness, sputum production Typical acute episode lasts up to several hours and responds to bronchodilators Some patients have persistence of mild symptoms between acute exacerbations Symptoms may be triggered by exposure to allergens, respiratory tract infections, exercise, medications, air pollution, cold, stress Physical exam reveals an increased respiratory rate and end-respiratory wheezing Spirometry shows a reduced FEV1/FVC ratio that improves with bronchodilators Radiologic Features Chest x-ray can appear normal or show hyperinflation or occasionally atelectasis Symptoms often disappear as children become adults Most patients with asthma can achieve symptomatic control and improvement of lifestyle through patient education, avoidance of potential triggers, and pharmacologic

therapy with antiinflammatory agents and bronchodilators Pathologic Features Gross Findings In fatal status asthmaticus, there are areas of overinflation and atelectasis, and filling of bronchi and bronchioles with mucus Microscopic Findings Mixed inflammatory cell infiltrates (with eosinophils) in bronchi and bronchioles Smooth muscle hypertrophy in airways Goblet cell hyperplasia in bronchi, goblet cell metaplasia of bronchioles Enlarged bronchial submucosal glands Mucus plugging of airways with mixed inflammation including eosinophils and Charcot-Leyden crystals Epithelial cell injury and loss, reserve cell hyperplasia, squamous Thickened bronchial basement membranes Modified from Green L: Emphysema and diseases of large airways. In Zander DS, Farver CF: Pulmonary Pathology, Elsevier, 2008.

83. Asthma: Histologic findings in asthma are those of chronic bronchitis, and in addition, airway smooth muscle hypertrophy and bronchial eosinophil infiltration. This example demonstrates a mixed eosinophil and chronic inflammatory infiltrate with overlying squamous metaplasia.

TYPES OF ASTHMA Extrinsic: 1. Atopic (allergic) 2. Occupational 3. Allergic bronchopulmonary aspergillosis Intrinsic: 1. Nonatopic 2. Drug-induced

PRECIPITATING FACTORS specific allergens chemicals, fumes, dusts, gases Aspergillus spores

MECHANISMS type I hypersensitivity reaction type I hypersensitivity reaction type I and III hypersens. reactions hyperreactive airways dec prostaglandins, inc leukotrienes

respiratory infection, esp. viral aspirin

Atopic Most common type of asthma, usually beginning in childhood Often have a family history of allergies IgE is usually elevated; inciting antigen(s) will usually provoke a positive (whealand-flare) skin test Type I hypersensitivity reaction: antigen reacts with sensitized mast cells on the mucosal surfaces, provokes release of mediators (histamine, ECF, NCF, leukotrienes, PGD2, PAF, etc.), initiates a Th2 T-cell response, and directly stimulates subepithelial vagal receptors ----> bronchoconstriction, edema, mucus secretion, flushing, epithelial damage (due to the major basic protein of eosinophils) Release of growth factors (TGF-beta, IGF, PDGF) and other substances will eventually cause airway fibrosis if the inflammatory reaction proceeds unchecked, so treatment is aimed at reducing inflammation as well as bronchoconstriction.

Nonatopic - inflammation of the respiratory mucosa may render subepithelial vagal receptors more sensitive to irritants ----> bronchoconstriction Allergic bronchopulmonary aspergillosis Bronchial colonization by Aspergillus leads to development of IgE and IgG antibodies against the fungus Exposure to spores leads to mast cell degranulation ----> bronchoconstriction and increased vascular permeability IgG antibodies form immune complexes with fungal antigens in bronchi ----> inflammation and fibrosis of bronchi

Churg-Strauss Syndrome (Allergic Angiitis and Granulomatosis) This syndrome develops in a small subset of people with asthma (and rare people without asthma), and is a systemic vasculitis that often involves the lungs, heart, and peripheral nerves. Patients have peripheral eosinophilia > 10% of white blood cells. Pathologic features: necrotizing vasculitis, tissue infiltration by eosinophils, extravascular granuloma
84. Churg-Strauss syndrome: Necrotizing vasculitis (arrow) with extravascular eosinophils (hard to discern at this power) is present in an appendix from a patient with asthma, peripheral blood eosinophilia, migratory pulmonary infiltrates, sinusitis.

Manifestations:

Pulmonary asthma, lung infiltration by eosinophils (chronic eosinophilic pneumonia), lung infiltrates caused by granulomas and vasculitis Cardiac heart failure, pericarditis, etc. Peripheral nerves neuropathy Gastrointestinal abdominal pain, diarrhea, etc. Renal insufficiency Skin nodules, purpura, urticaria, etc.

BRONCHIECTASIS
Chronic necrotizing infection, usually with mixed flora, of bronchi and bronchioles, with abnormal airway dilatation Pathogenesis: Airway obstruction and infection are commonly part of the pathogenesis. Severe, often longstanding inflammation permanently damages bronchial wall components, which are eventually replaced with scar tissue. Airway obstruction may predispose to infection or result from infection due to increased luminal secretions and exudates, which also contribute to dilatation. Associated with production of large amounts of purulent sputum particularly in the morning, when patients arise and the purulent material drains

Associated conditions: Bronchial obstruction (tumor, mucus impaction, foreign body) ----> usually localized bronchiectasis, rarely diffuse (in atopic asthma or chronic bronchitis) Necrotizing infectious pneumonias: bacterial, tuberculous, adenoviral, influenza-associated, Aspergillus (ABPA) Congenital or hereditary conditions: o Immotile cilia and Kartagener's syndromes (bronchiectasis, sinusitis, situs inversus): structurally abnormal cilia, often with absent or irregular dynein arms, defective motility Poor bacterial clearance in lungs and sinuses ----> chronic infections Abnormal cell motility during embryogenesis ----> situs inversus

o Cystic fibrosis and immunodeficiency states ----> chronic infections Other chronic inflammatory conditions: lung transplant rejection, graft-versushost disease, collagen vascular diseases

Morphology: Gross: marked dilatation of airways, with airways visible to within 2-3 cm. of pleura Microscopic: variable acute and chronic inflammation, ulceration, necrosis, abscess formation, squamous metaplasia, and fibrosis

85. Bronchiectasis, localized: Bronchi (arrow) obstructed by a tan tumor (center) are markedly dilated distal to the tumor.

86. Bronchiectasis, generalized: All bronchi are markedly dilated. Bronchial mucosae are yellow-tan and ragged, reflecting the presence of an ongoing suppurative infection. Proximal bronchial walls are pale and fibrotic, due to longstanding infection and subsequent scarring.

87. Bronchiectasis: The bronchial wall is markedly inflamed. The epithelium may be intact, as it is here, or it may be ulcerated.

TUMORS OF THE LUNG


Lung Cancer
Leading cause of cancer death in the U.S. During the last 50 years, the number of deaths/yr. due to lung cancer increased markedly in the U.S.: 18,000 in 1950 vs. 153,000 in 1994 The death rate for men has leveled off; for women, the rate continues to rise; changes trail behind decreases in the prevalence of cigarette smoking Peak incidence in 50s and 60s; occurs most often between ages 40 and 70

Etiology/Pathogenesis Tobacco smoking Statistical evidence from retrospective studies: association between the frequency of lung cancer and the number of cigarettes smoked/day, duration of smoking, and the tendency to inhale o 87% of lung cancers occur in smokers; smoking is also associated with ENT cancers and cancers of the esophagus, bladder, pancreas o Avg. smokers have 10X risk of nonsmokers, heavy (>40/day) >20X risk o After approx. 10 - 15 years without cigarettes, risk approaches baseline for light smokers (< 1 pack/day), but remains elevated (4X) for heavy smokers o Cigar and pipe smoking increase risk, but not as high as cigarettes Clinical evidence: increased atypical and hyperplastic changes in the tracheobronchial tree of smokers (96.7%) vs. controls (0.9%) Experimental evidence: o Of the more than 1200 substances present in cigarette smoke, many are potential carcinogens - initiators (polycyclic aromatic hydrocarbons), promoters (phenol derivatives), radioactive elements, other (arsenic, nickel, molds, etc.) o Benzopyrene, a component of cigarette smoke, causes DNA damage at lung cancer mutational hotspots in p53 Industrial hazards numerous reported, including Radiation - uranium miners have increased risk Asbestos exposure - increases risk, especially in combination with smoking

Air pollution

Small "urban factor" of increased risk Radon is linked to lung cancer in miners exposed to high concentrations, but risk of lower concentrations is not clearly defined

Genetics Dominant oncogene mutations often involved in lung cancer: c-MYC (esp. small cell ca), K-RAS (esp. non-small cell ca), EGFR, HER2neu Commonly deleted or inactivated tumor suppressor genes include p53 (both small cell and non-small cell carcinomas), RB (esp. small cell ca), p16 (esp. nonsmall cell ca), and multiple loci on chromosome 3p Some oncogene mutations and deletions of tumor suppressor genes can also be found in morphologically benign lung epithelial cells, probably due to a field effect Occasional familial groupings; polymorphisms in cytochrome P450 gene CYP1A1 have been identified as markers for genetic susceptibility Variable risk among smokers of developing lung cancer

Scarring - usually a desmoplastic reaction to a cancer, but occasionally precedes a cancer (usually an adenocarcinoma); some diffuse fibrosing conditions (ex: asbestos, scleroderma, idiopathic pulmonary fibrosis) are associated with an increased risk of bronchogenic carcinoma Precursor Lesions (and their associated cancers) Squamous dysplasia and carcinoma in situ (squamous cell carcinoma) Atypical adenomatous hyperplasia (bronchioloalveolar carcinoma) Diffuse idiopathic pulmonary neuroendocrine cell hyperplasia (carcinoid)

Note: These lesions can (but do not always!) lead to the associated cancer.

Major Histologic Types of Malignant Epithelial Lung Tumors Adenocarcinoma: 25-40% (most common) Squamous cell carcinoma: 25-40% Small cell carcinoma: 20-25% Large cell carcinoma: 10-15% Carcinomas with sarcomatoid elements Carcinoid Carcinomas of salivary gland type Other tumors Non-small cell carcinoma: this term is used to include squamous cell carcinoma, adenocarcinoma, and large cell carcinoma, as well as some less common types of lung cancer, but NOT small cell carcinomas. Mixed types of carcinomas (e.g., adenosquamous carcinoma, combined small cell and squamous cell carcinoma, etc) are not uncommon (~10%). Staging: The TNM system is used to stage bronchogenic carcinomas. This system incorporates the data for tumor size, extension to adjacent structures, nodal metastases, and distant metastases, to arrive at a stage (I - IV). Prognosis generally worsens with higher stages.

Adenocarcinoma
Males and females equally affected Less associated with smoking than squamous or small cell carcinomas, but most patients (>75%) have a history of smoking Most common type of lung cancer in women and nonsmokers Usually peripherally located; may be associated with a scar Morphology: o Gland formation is present, with complex glands, back-to-back glands, cribriforming, papillary processes, solid areas; more well differentiated tumors have more gland formation, and poorly differentiated tumors are more solid o Columnar or cuboidal cells with pleomorphic nuclei, often large nucleoli o 80% contain mucin EGFR mutations are found in some of these tumors, more often in bronchioloalveolar carcinomas K-RAS mutations are found in some of these tumors, more often in smokers (30%) than non-smokers (5%)

88. Adenocarcinoma: This type of neoplasm characteristically occurs in the periphery of the lung.

89. Adenocarcinoma: This tumor forms complex glandular arrangements. Glands are focally back-to-back. Mucinous secretions are present in glandular lumens.

90. Adenocarcinoma: As in this example, tumor cells are often columnar, with variably hyperchromatic and pleomorphic nuclei and prominent nucleoli.

91. Adenocarcinoma: This mucicarmine stain reveals intracellular mucin (red-pink), which indicates glandular differentiation and is a marker for adenocarcinomas.

Bronchioloalveolar Carcinoma A subtype of adenocarcinoma, currently defined as an in situ adenocarcinoma Accounts for 1-9% of lung cancers; increasing incidence for unclear reasons Usually peripherally located, probably arise in terminal bronchioles or alveoli Solitary or multiple nodules, or resembles pneumonia on CXR; believed to spread aerogenously Overall 5 year survival rate is about 25%, better prognosis than other common types of lung cancer; surgically resectable solitary lesions have 5 year survival rate of 50-75%; slower growing than other types of lung cancer Morphology: tall columnar to cuboidal epithelial cells (differentiation along lines of mucin-secreting bronchiolar cells, Clara cells, and/or type II pneumocytes) that line alveolar septa, may form papillary projections; most tumors are welldifferentiated

92. Bronchioloalveolar carcinoma: Bronchioloalveolar carcinoma may present as a single or multiple discrete nodules, or as in this slide, in a manner resembling pneumonia.

93-94. Bronchioloalveolar carcinoma: This type of malignancy is defined by its pattern of growth: it spreads along alveolar septa without disrupting them, using them as its stroma. Most of these tumors are well differentiated, and composed of columnar or cuboidal epithelial cells that can contain mucin.

Squamous Cell Carcinoma


Most common in men, highly associated with smoking Most arise near the hilum, in the first-third order bronchi, and are often associated with dysplasia and carcinoma in situ Thickening and irregularity of the bronchial mucosa may be seen with a bronchoscope Well differentiated forms have prominent keratin production and intercellular bridges; poorly differentiated tumors may have only focal keratin, and look almost undifferentiated Often have prominent necrosis and may cavitate Tend to spread locally and metastasize later than other patterns, but growth rate is usually more rapid than other histologic types Highest frequency of p53 mutations among the lung cancers

95. Squamous cell carcinoma: This warty exophytic tumor (box) originated in a proximal bronchus and has only a small invasive component (not visible). 96. Squamous cell carcinoma: This large tumor is cavitated, a consequence of necrosis.

99. Carcinoma in situ: Nuclear pleomorphism and hyperchromasia exist throughout the full thickness of the epithelium. Lack of epithelial maturation is evident. The process remains superficial to the basement membrane (in situ).

97-98. Squamous cell carcinoma, well differentiated: Large nests of keratinizing epithelial cells with large hyperchromatic pleomorphic nuclei comprise this tumor. The round aggregates of keratin are "keratin pearls". The stroma is desmoplastic, reflecting tumor invasion.

100. Squamous cell carcinoma with post-obstructive pneumonia: Distal to the tumor, the lung is pale and firm (consolidated), corresponding to an acute pneumonia.

Small Cell Carcinoma


More common in men, strong association with smoking; most aggressive type of lung cancer, demonstrates early and widespread metastasis Usually centrally located (near hilum) Small cells with high nuclear:cytoplasmic ratios, fine chromatin, small or absent nucleoli, little cytoplasm; "crush artifact" often seen Often show neuroendocrine differentiation by immunohistochemistry (+ synaptophysin, chromogranin) and EM (dense core neurosecretory granules) P53 and RB tumor suppressor genes are commonly mutated in these tumors

101. Small cell carcinoma with hilar lymph node metastases: The tan tumor (within curved line) arose from the contiguous bronchus and metastasized to multiple hilar lymph nodes (arrows).

102-103. Small cell carcinoma: The tumor consists of small cells with small nuclei and high nuclear:cytoplasmic ratios (little cytoplasm). The chromatin has a fine salt and pepper appearance, without nucleoli.

Large Cell Carcinoma


Large cells with no light microscopic evidence of squamous or glandular differentiation

Clinical Course
Symptoms: cough, weight loss, chest pain, dyspnea CXR: lung mass or infiltrate Local metastasis to regional lymph nodes in about 50% of cases Distant metastasis occurs via lymphatic or hematogenous routes; common sites are the adrenals (>50% of cases), liver, brain, bone ~15% overall 5 year survival rate Tumor stage is the most important factor in predicting survival; unfortunately, many cancers are discovered after they have become unresectable or metastasized Small cell carcinomas are not usually treated surgically, but are usually sensitive to radiation and chemotherapy; 1 year mean survival after diagnosis Non-small cell carcinomas are usually treated surgically, unless metastases have occurred or the tumor is unresectable; localized tumors < 4 cm. diameter treated surgically ----> 5 year survival rate of up to 40% for squamous cell ca and up to 30% for adenoca and large cell ca

Local Phenomena Associated with Lung Cancer


Bronchial obstruction bronchitis, pneumonia, bronchiectasis, abscess, lipoid pneumonia Compression or invasion of the superior vena cava SVC syndrome Extension through pleura or pericardium pleuritis, pericarditis, effusions Extension into or around esophagus dysphagia Extension into or around recurrent laryngeal nerve hoarseness Extension into or around phrenic nerve diaphragmatic paralysis Extension into chest wall rib destruction

Some paraneoplastic syndromes


ADH SIADH; esp. small cell ACTH Cushing's syndrome; esp. small cell Parathormone (PTH), PTH-related peptide, prostaglandin E hypercalcemia; esp. squamous cell Serotonin carcinoid syndrome; esp. carcinoids or rarely small cell carcinoma

Some systemic manifestations


Lambert-Eaton myasthenic syndrome - muscle weakness caused by autoantibodies reactive with the neuronal calcium channel Hypertrophic pulmonary osteoarthropathy - finger clubbing Venous thrombosis and embolism, nonbacterial thrombotic endocarditis - due to hypercoagulable tendency particularly associated with adenocarcinomas

Pancoast's tumors
Located in the apical region of the lung, in the superior pulmonary sulcus May invade the cervical sympathetic plexus ----> Horner's syndrome (enopthalmos, ptosis, miosis, anhidrosis), and the brachial plexus ----> pain in the distribution of the ulnar nerve

Carcinoid (not a type of pulmonary carcinoma)


~1-5% of lung tumors; most patients are < 40 years old Not related to smoking or any other recognized environmental factors Most < 4 cm. in diameter; most grow intraluminally in the major bronchi, and are associated with pneumonia, bronchiectasis, emphysema, and atelectasis; some invade the bronchial wall and fan out into the peribronchial tissues ("collar button") 5-10% metastasize to regional lymph nodes; 5 year survival rate is 90-95% Occasionally secretory, and associated with the carcinoid syndrome intermittent diarrhea, flushing, cyanosis Occasionally found in patients with multiple endocrine neoplasia type I Morphology Cells have uniform round nuclei and infrequent (<2/10hpf) mitoses, in nests and cords, separated by delicate fibrovascular stroma Neuroendocrine differentiation (like Kulchitsky cells) evident by EM (dense core neurosecretory granules in the cytoplasm) or immunohistochemistry (stain for chromogranin, synaptophysin, other peptides)

Atypical carcinoids - increased nuclear pleomorphism, increased mitoses, and necrosis; have a more aggressive course (50% recurrence or metastasis after 2 years)

104. Carcinoid: A polypoid intraluminal mass (arrow) with a smooth surface, in a bronchus is a common presentation.

105. Carcinoid: Tumor cells form nests, and typically have uniform nuclei with little pleomorphism, few mitoses, and a fibrovascular stroma. The presence of necrosis, increased mitoses, and increased nuclear pleomorphism is associated with a worse prognosis (lower survival, higher frequency of metastasis).

106. Carcinoid: This chromogranin stain is positive (brown), indicating neuroendocrine differentiation (as one would expect with this tumor). Chromogranins are located in neurosecretory granules.

Hamartomas (Benign Mesenchymomas, Fibrochondrolipomas)


Most common benign tumors of the lung CXR and CT scans lung nodule, central or peripheral; may see low density areas that correspond to fat; may have calcifications Symptoms usually asymptomatic, but if central, can be polypoid and associated with symptoms of airway obstruction (recurrent pneumonia, atelectasis) Morphology: Gross: well-circumscribed, lobulated nodule that pops out of lung Microscopic: benign cartilage, fibrous tissue, adipose tissue

107. Hamartoma: This well-circumscribed tanwhite nodule has a lobulated appearance and separates easily from the adjacent lung tissues.

108. Hamartoma: A combination of benign hyaline cartilage, adipose tissue, and fibrous tissue is present. As shown, clefted infoldings lined by respiratory epithelium are commonly seen.

Metastatic tumors
Spread by lymphatics, blood vessels, or contiguity from the mediastinum or esophagus Often present with multiple nodules in multiple lobes A second relatively common pattern is that of growth in peribronchiolar and perivascular tissue spaces (secondary to lymphatic spread); septa may appear prominent or tumor may be grossly invisible, but microscopically present in lymphatics

109. Metastatic carcinoma: This is the "cannonball" (multinodular) pattern of metastasis.

110. Metastatic carcinoma: Lymphangitic spread of tumor is shown here (*). Two lymphatics are filled with fragments of tumor. Tumor spread via this route may be extensive within the lungs.

PLEURAL DISEASES
Pleural effusions (> 15 ml. of serous fluid) develop in a variety of settings.
Identification as a transudate or an exudate helps to identify potential causes. PF protein (g/dL) TRANSUDATE EXUDATE < 3.0 > 3.0 PF protein/S protein < 0.5 > 0.5 PF LD (IU) < 200 > 200

PF LD/S LD < 0.6 > 0.6

PF = pleural fluid, S = serum, LD = lactate dehydrogenase

MECHANISMS increased hydrostatic pressure decreased plasma oncotic pressure increased intrapleural negative pressure

EXAMPLES congestive heart failure nephrotic syndrome cirrhosis of the liver atelectasis infections - TB, pneumonia neoplasms - primary, met infarcts collagen vascular diseases neoplasms

TRANSUDATES (Hydrothorax)

increased vascular permeability EXUDATES decreased lymphatic resorption

Special types of effusions and their features: Chylous effusion - milky fluid of lymphatic origin, containing emulsified fats; usually associated with tumors that block and cause rupture of major lymphatic ducts Pancreatitis - elevated PF amylase Ruptured esophagus - elevated PF amylase, low pH Blood (hemothorax) - ruptured blood vessel (aortic aneurysm, trauma, etc.)

Pleuritis
Type of Pleuritis Serofibrinous Suppurative (empyema) Hemorrhagic Exudate Fibrinous Neutrophilic Etiologies Adjacent inflammatory processes in the lungs (infections, collagen vascular diseases), infarcts, uremia, radiation Pleural space infection usually secondary to pneumonia, occasionally from hematogenous infection or spread from below the diaphragm Neoplasms involving pleural space, hemorrhagic diatheses

Bloody

111. Fibrinous pleuritis: Fibrin deposition is present on the pleural surface, in this view from a patient with rheumatoid arthritis

Pneumothorax - air or gas in the pleural cavity


Etiologies Spontaneous Lung diseases that produce rupture of an alveolus, such as emphysema or abscess; idiopathic variant occurs in young people due to rupture of small subpleural blebs Perforating injury to the chest wall +/- lung Used to treat TB in the past

Traumatic Therapeutic

Tension pneumothorax: air enters the pleural cavity, usually during inspiration, but cannot escape during expiration ----> increased pressure ----> lung collapse, sometimes compression of mediastinal structures, and mediastinal shift (visible on CXR)

Pleural Tumors
Secondary involvement of the pleural space by extrapleural malignancies (especially lung and breast cancer) is more common than are primary pleural tumors. Primary pleural tumors include solitary fibrous tumors, malignant mesotheliomas, and other relatively rare entities. Solitary fibrous tumors are also rare and consist of fibroblast-like cells and collagen. Malignant mesotheliomas are uncommon and are linked to asbestos exposure. In people with significant asbestos exposure, however, bronchogenic carcinoma is much more common than mesothelioma. Malignant Mesothelioma Arise from either the visceral or parietal pleura Associated with heavy asbestos exposure: 7-10% lifetime risk, 25-45 year latent period; no increased risk with smoking (unlike bronchogenic carcinoma) Symptoms: chest pain, dyspnea, recurrent pleural effusions Very aggressive tumor with 50% mortality within one year of diagnosis Peritoneal mesotheliomas are extremely rare, and are associated with especially heavy asbestos exposure Morphology: o Gross: spreads extensively in the pleural space, surrounds and invades lung, invades the chest wall o Microscopic: epithelial (often resembling adenocarcinoma), sarcomatoid (resembling fibrosarcoma) and mixed types; immunohistochemical stains or electron microscopy (EM) is usually needed for diagnosis EM ----> long thin microvilli (vs adenoca - short microvilli) Immunohistochemical stains a panel approach is usually taken including multiple stains (+) in mesothelioma and (+) in the other diagnostic considerations

114. Malignant mesothelioma, desmoplastic: Cytologically bland tumor cells have a spindle shape and lie in a collagenous background.

112. Malignant mesothelioma: This aggressive neoplasm has grown to fill the pleural space and compress the lung.

115. Malignant mesothelioma, biphasic: Epithelial (right) and sarcomatoid (left) tumor cells are present.

113. Malignant mesothelioma, epithelial: Tumor cells have an epithelial appearance polygonal cells with abundant cytoplasm and round nuclei.

116. Lung carcinoma in pleural space: Lung cancers commonly invade pleura and disseminate within the pleural space.

Case Histories and Images


Case #1: A 65-year-old man experiences shortness of breath that lasts for several hours and goes away. Several months later, he has another similar episode, but the shortness of breath is followed by localized chest pain on his right side. Physical examination reveals a pleural friction rub and thrombosis of his right posterior tibial vein.

Case #1, Image #2

Case #1, Image #1

Case #2: A 9-month-old girl has difficulty breathing and a fever. A chest x-ray shows patchy bilateral infiltrates.

Case #2

Case #3: A 70-year-old man who worked with asbestos for several decades notices difficulty breathing and reduced ability to exercise. His chest x-ray shows diffuse bilateral shadows and a 4 cm lung mass.
Case #3, Image #2. Iron stain on right.

Case #3, Image #1 Case #3, Image #3

Case #4: For the last three years, a young woman has boarded her brothers parakeet at her apartment for several weeks every year. She has noticed that when the bird stays with her, she becomes progressively more short of breath, and the shortness of breath persists for weeks after the bird has gone back to his normal abode. With time, however, the shortness of breath disappears, only to return when the bird returns.

Case #4, Image #1

Case #4, Image #2

Case #5: A 33-year-old binge drinker has a 3 cm. cavitary mass on his chest xray, and produces large amounts of foul-smelling sputum.

Case #5, Image #2 Case #5, Image #1

Case #6: A 43-year-old HIV-infected patient has apical infiltrates in both lungs, and a pleural effusion.

Case #6, Image #1

Case #6, Image #2. Acid fast stain.

Case #7: A 52-year-old lung transplant recipient develops a fever and patchy bilateral lung infiltrates on chest x-ray.

Case #7

Case #8: An 80-year-old man with congestive heart failure develops lung infiltrates that are predominantly basilar in distribution, and a pleural effusion. The pleural fluid protein is 2.0 g/dL, and the LD is 100 IU.

Case #8

Case #9: A 68-year-old man has been a heavy smoker for 30 years. He has noticed worsening shortness of breath.

Case #9

Case #10: A 6-year-old girl wheezes whenever she plays with her friends cat.

Case #10

Case #11: A 34-year-old asymptomatic woman gets a chest x-ray, which, surprisingly, shows enlarged mediastinal and hilar lymph nodes, and some faint lung infiltrates.

Case #11

Case #12: A 24-year-old man has had several episodes of pneumonia over the last 2 years. A chest x-ray shows a well-circumscribed 2 cm. nodule in his right upper lobe. Questioning also reveals that he has had some diarrhea and flushing over the last several months.

Case #12, Image #2

Case #12, Image #1

Case #13: A 57-year-old man with esophageal cancer, who is receiving radiation therapy, develops a cough, fevers, and a left lower lobe infiltrate.

Case #13 Case 14. Silver methenamine stain.

Case #14: A 50-year-old woman has chest discomfort. A chest x-ray shows a peripheral infiltrate.

Self-Quiz Questions
Vascular 1. Hemodynamic causes of pulmonary edema include all of the following except: a. increased hydrostatic pressure b. lymphatic obstruction c. decreased oncotic pressure d. microvascular injury 2. Which of the above categories of mechanisms (a-d) are responsible for the development of pulmonary edema in a. congestive heart failure? b. nephrotic syndrome? c. lymphangitic carcinomatosis? d. pneumonia? 3. Which statement is true about pulmonary embolism? a. The physiologic consequences are unrelated to the extent of pulmonary vascular obstruction. b. Large pelvic veins are the most common source of pulmonary emboli. c. The status of collateral (bronchial) circulation is a major determinant of the significance of an embolus. d. Pulmonary emboli frequently lead to compromise of left heart function. 4. Disorders associated with an increased incidence of pulmonary emboli include all of the following except: a. cancer b. factor V Leiden c. chronic immobilization d. hemophilia Which of the following clinical consequences of emboli is 5. Most common? 6. Least common? a. sudden death b. nothing c. pulmonary infarction d. pulmonary hemorrhage 7. A radiodense area appears on a patient's chest X ray. Physical examination reveals a tender area in the calf, with a rope-like consistency. Two days later, the patient dies suddenly. Which histologic findings might you expect to see in the lungs? a. ischemic necrosis of the lung parenchyma with hemosiderin-laden macrophages b. intraalveolar neutrophils c. interstitial and intraalveolar fibrosis d. small arteries with intimal proliferation and fibrosis 8. A 20 year old intravenous drug abuser develops a new cardiac murmur. Several days later, he complains of shortness of breath and dyspnea. A chest X ray will most likely show/pathologic examination of the lungs may show (best answer): a. central nodular infiltrate/fibrin thromboembolus with fungal hyphae and neutrophils b. central nodular infiltrate/intimal fibrosis in small arteries c. wedge-shaped peripheral infiltrate/fibrin thromboembolus with fungal hyphae and neutrophils d. wedge shaped peripheral infiltrate/fat thromboembolus 9. Pulmonary hypertension is a. a consequence of decreased pulmonary vascular resistance b. linked to mutations in the bone morphogenetic protein receptor type 2 (BMPR2) signaling pathway, in many cases of primary pulmonary hypertension c. usually primary, without associated cardiopulmonary abnormalities d. reflected by development of atheromatous lesions in large pulmonary veins 10. Pulmonary veno-occlusive disease a. Is a type of vasculitis. b. Is a congenital anomaly. c. can be treated by lung transplantation. d. usually responds to corticosteroid therapy. 11. Which of the following statements is true? a. Idiopathic pulmonary hemosiderosis is associated with vasculitis. b. Antineutrophil cytoplasmic antibodies are associated with Wegeners granulomatosis. c. Linear deposition of IgG is found in Behcet disease. d. The kidneys are usually spared in Goodpasture Syndrome. 12. Substances thought to be involved in the pathogenesis of ARDS include all of the following except: a. eosinophil major basic protein b. oxygen-derived free radicals c. proteases d. thromboxane 13. In the acute stage of ARDS a. interstitial fibrosis is present b. ventilation-perfusion mismatch is present c. the lung is hyperinflated d. mechanical ventilation is not required 14. Histologic features of the acute stage of ARDS include all of the following except: a. hyaline membranes b. edema c. sloughed epithelial cells d. honeycomb change 15. Which statement is incorrect? a. ARDS develops slowly, and can be circumvented if therapy is instituted quickly. b. The chest X ray in ARDS shows diffuse pulmonary infiltrates. c. The outcome of ARDS may be resolution with variable residual lung damage, or death. d. The mortality rate of ARDS in the U.S. is about 60%.

Bacterial infections 1. Protective mechanisms which keep the lung free of significant numbers of organisms include all of the following except: a. phagocytosis by alveolar macrophages b. mucociliary action of the lower respiratory passages c. filtering by nasopharynx d. release of destructive enzymes by pneumocytes 2. Factors predisposing to development of nosocomial infections include all of the following except: a. antibiotic resistance of hospital-associated organisms b. bacterial colonization of respirator apparatus c. frequent washing of hands d. lowering of host defenses due to illness 3. Bronchopneumonia and lobar pneumonia differ in that: a. they are caused by different organisms b. the pattern and extent of involvement of the lung are usually different c. lobar pneumonia is commoner than bronchopneumonia d. neutrophils are not a part of the host response with lobar pneumonia 4. The most common organism producing lobar pneumonia is: a. Strep. pneumoniae b. Staph. aureus c. Klebsiella pneumoniae d. Pseudomonas aeruginosa 5. Complications of pneumonia include all of the following except: a. abscess formation b. bacteremia c. bronchitis d. pulmonary fibrosis 6. You are invited to observe an autopsy. The lungs are removed, and appear red, firm, and airless. The principal histologic findings you would expect are: a. intraalveolar fluid and bacteria b. neutrophils and red cells in alveoli c. intraalveolar fibrin and disintegrating white cells d. intraalveolar fibrosis 7. Conditions associated with an increased risk of aspiration pneumonia include all of the following except: a. dieting b. unconsciousness c. tumor involving the epiglottis d. gastroesophageal reflux 8. A comatose patient is observed to aspirate while lying on his back. If he develops an abscess, it will most likely be in which region, and contain which organisms? a. perihilar/Strep pneumoniae b. posterior region of upper lobe/anaerobes c. apical segment of right lower lobe/anaerobes d. none of the above 9. A common cause of hematogenous pneumonia is: a. Strep. pneumoniae b. Pseudomonas aeruginosa c. Pneumocystis d. Histoplasma capsulatum

10. Which of the following statements regarding Legionella


pneumophila is correct? a. the bacteria is easily seen in tissues with a gram stain b. special culture media is required for isolation of the organism c. the bacteria typically causes a childhood illness d. the host response to the organism is granulomatous 11. A 65 year old patient complains of a cough that produces abundant foul-smelling sputum. A chest x-ray shows a mass with an air-fluid level. Which of the following is least likely to be correct: a. The lung lesion may be related to the patients tooth extraction 2 months ago. b. A previous septic embolus may have prompted development of the lung lesion. c. Histologic sections of the lesion will show sheets of macrophages. d. Concern about a neoplasm is justified. 12. Which of the following correctly pairs a clinical presentation with an organism and a characteristic? a. Atypical pneumonia Actinomyces gram + b. Acute pneumonia Histoplasma GMS + c. Acute pneumonia cytomegalovirus gram + d. Chronic pneumonia Nocardia acid fast + Atypical Pneumonias 1. Clinical manifestations of viral infection are influenced by all of the following except: a. age b. immune status c. coexisting illness d. history regarding previous exposure to the virus e. travel history The most common cause of serious lower respiratory tract infection in children less than 1 year old is a. influenza A b. respiratory syncytial virus c. measles d. varicella zoster virus A common viral cause of pneumonia in military recruits is a. herpes simplex virus b. cytomegalovirus c. respiratory syncytial virus d. adenovirus Healthy children are least likely to develop a severe pneumonia caused by a. herpes simplex virus b. influenza A c. parainfluenza d. respiratory syncytial virus Histologic patterns of reaction to viral pathogens and mycoplasma include all of the following except: a. diffuse alveolar damage b. bronchiolitis c. interstitial pneumonia d. emphysema

2.

3.

4.

5.

6.

Multinucleated giant cells are a feature of infections by all of the following viruses except: a. influenza A b. measles c. respiratory syncytial virus d. herpes simplex virus "Smudge cells" are characteristic of a. cytomegalovirus a. herpes simplex virus b. adenovirus c. measles Which statement is incorrect? a. HSV produces ulcerative tracheobronchitis and hemorrhagic parenchymal nodules. b. Antigenic shifts precede influenza pandemics. c. The neuraminidase antigen is believed to mediate entry of the influenza virus into host cells. d. Mycoplasma pneumoniae usually causes a mild communityacquired pneumonia and often triggers production of cold agglutinins. Which statement is incorrect? a. CMV pneumonia occurs almost exclusively in immunocompromised patients and neonates b. Pregnancy does not influence the prognosis of varicella pneumonia. c. Pneumonia is uncommon in kids with chicken pox. d. Pneumocystis cysts can be seen with a silver stain

5. Postprimary tuberculosis a. typically involves the apical segments of the upper lobes. b. has the histologic features of acute bronchopneumonia. c. is usually caused by reinfection by exogenous bacilli. d. does not require the presence of viable acid bacilli. 6. Tuberculous cavities a. develop within inactive calcified granulomas. b. harbor numerous bacilli which can frequently be detected in a sputum sample. c. do not communicate with an airway, preventing transmission of tuberculosis. d. are actually enlarged vascular spaces. 7. Which statement about extrapulmonary tuberculosis is incorrect? a. It usually occurs as a consequence of lymphatic or vascular spread of the organisms. b. It is more likely to develop in HIV-positive patients, than in HIV-negative people. c. It is not usually associated with pulmonary tuberculosis. d. Impairment of an infected organ may occur. 8. Which statement about tuberculosis and HIV infection is incorrect? a. The risk of development of active disease in a PPD + patient is about 8% per year. b. Effects of HIV on T cells and macrophages may impair their ability to respond to mycobacteria. c. In a lung biopsy, tuberculosis is not present if macrophages infiltrates do not form granulomas. d. Tuberculosis often occurs relatively early in the course of HIV infection. 9. Atypical mycobacterial infections a. occur in elderly patients with chronic lung diseases. b. usually are transmitted between people.

7.

8.

9.

Tuberculosis 1. Which of the following statements is incorrect? a. The incidence of tuberculosis in the U.S. has increased since 1985. b. Patients with AIDS and the elderly are the two populations most often presenting with active disease. c. Speaking and sneezing can release viable acid fast bacilli into the atmosphere.

c. are caused by Mycobacterium tuberculosis.


d. are easily treated. 10. Matching - choose the response from the column on the right which matches with each item in the column on the left. a. primary Tb ____ m. bronchial nodules, ulcers b. fibrocalcific TB ____ n. calcified apical granulomas c. cavitary TB ____ o. Ghon complex d. tracheobronchial TB ____ p. lobar consolidation e. TB pneumonia ____ q. vertebral TB osteomyelitis f. TB pleuritis ____ r. granuloma that eroded into a bronchus and drained g. miliary TB ____ s. innumerable small lesions h. Pott's disease ____ t. thickened pleura with associated pleural effusion

d. Infection with Mycobacterium tuberculosis is always


associated with clinical disease.

2. Mycobacterium tuberculosis
a. is a gram positive rod. b. has a highly permeable coat, allowing for good penetration of antimicrobial drugs. c. may be visualized with a Ziehl-Neelsen stain. d. is quickly killed by macrophages, due to the presence of "cord factor". 3. Important components of the immune response to tuberculosis are thought to include all of the following except: a. B cells. b. CD4+ T cells. c. macrophages. d. CD8+ T cells. 4. A 1.0 cm. subpleural focus of parenchymal consolidation is found in the upper portion of the left lower lobe of the lung during autopsy. This lesion contains acid fast bacilli. Which form of tuberculosis does this focus most likely represent? a. reactivation tuberculosis. b. endobronchial tuberculosis. c. miliary tuberculosis.

d. primary tuberculosis.

Fungal infections 1. Which statement about fungal infections of the lungs is true? a. Fungi are not pathogenic for healthy people. b. Definitive identification of a fungus can be made by researching the patient's travel history. c. PAS and methenamine silver stains are routinely used to look for fungi in tissues. d. The immune response to fungal infections is not influenced by host immune status. 2. An encapsulated yeast that stains red with the mucicarmine stain is a. Cryptococcus. b. Histoplasma. c. Aspergillus. d. Rhizopus. 3. Morphologic features of fungi important in their identification include all of the following except a. presence or absence of hyphal septation. b. budding characteristics. c. hyphal branching characteristics. d. yeast size. e. cytopathic effects. 4. Histoplasma a. is acquired via ingestion of dust from soil contaminated with bat droppings. b. is chronic and progressive in most people. c. produces localized masses in the lungs which can be mistaken for neoplasms. d. is an intracellular parasite of lymphocytes. 5. Necrotizing granulomas are associated with all of the following infections except: a. histoplasmosis. b. cryptococcosis. c. tuberculosis. d. Pseudomonas pneumonia. 6. Cryptococcus a. does not cause disease in healthy people. b. can be diagnosed with the India ink test. c. demonstrates broad aseptate hyphae which branch at right angles. d. is acquired via inhalation of cow dung. 7. Which statement about mucormycosis is incorrect? a. Rhinocerebral mucormycosis occurs in patients with diabetic ketoacidosis. b. Organisms causing mucormycosis are "breadmold fungi": Rhizopus, Mucor, Absidia. c. This infection responds quickly to medical antifungal therapy. d. The organisms frequently invade blood vessels, causing thrombosis and subsequent infarction. Matching - Choose an answer (A-C) for questions 8-10. 8. colonization of bronchial mucosa assoc. with worsening asthma 9. necrotizing pneumonia in an immunocompromised patient 10. fungus ball developing in a preexisting cavity A. aspergilloma B. invasive aspergillosis C. allergic bronchopulmonary aspergillosis

Interstitial lung diseases 1. Features common to diffuse interstitial diseases of the lung include all of the following except: a. patients have dyspnea and tachypnea, and may eventually develop hypoxemia and cyanosis b. pulmonary hypertension may develop due to destruction of the pulmonary capillary bed c. produce obstructive functional alterations in pulmonary function d. typically bilateral 2. The causes of interstitial lung disease a. include a variety of environmental antigens b. can be identified in all cases c. are the same for everyone d. respond to antibiotics 3. Participants in the pathogenesis of interstitial lung disease include all of the following except: a. inflammatory mediators b. neutrophils c. macrophages d. fibroblasts e. none of the above 4. Honeycomb lung a. refers to the replacement of alveoli by cystic spaces separated by thick fibrous bands with inflammatory cells b. refers to the enzymatic destruction of alveolar septa and creation of cystic spaces c. refers to patchy intraalveolar fibrosis occurring after bacterial pneumonia d. is manufactured by Kelloggs 5. Which statement is incorrect? Sarcoidosis a. may be associated with cutaneous anergy b. is a systemic disease of unknown etiology c. rarely responds to corticosteroid therapy d. is associated with increased CD4+ T-cells in the lungs, with oligoclonal expansion of T-cell subsets 6. In sarcoidosis, which of the following structures may be seen in granulomas? a. asteroid bodies b. acid fast bacilli c. fungi d. talc 7. Farmer's lung is a type of a. localized lung disease b. idiopathic pulmonary fibrosis c. hypersensitivity pneumonitis d. infectious lung disease 8. A lung biopsy shows interstitial pneumonitis, chronic bronchiolitis, and granulomas. Treatment for this process usually includes a. antibiotics b. cytotoxic chemotherapy c. identification and avoidance of the etiologic agent d. surgery to remove the affected portion of lung

9. Clinical characteristics of idiopathic pulmonary fibrosis include all of the following except: a. usually steroid responsive b. insidious onset of dyspnea c. survival after diagnosis is less than 5 years d. restrictive functional alterations in pulmonary function 10. Histologic features of idiopathic interstitial fibrosis a. are also seen with other disorders b. include numerous hyaline membranes in the acute stage c. include interstitial fibrosis and granulomas d. reflect the initiation of the process by a stimulus at a single point in time 11. Cryptogenic organizing pneumonia a. is irreversible b. demonstrates numerous neutrophils in bronchioles c. is often preceded by symptoms of gastroenteritis d. has pathologic features that can be similar to infections, aspiration, or collagen vascular diseases 12. Pneumoconioses may be associated with inhalation of a. silica. b. iron oxide. c. beryllium. d. coal dust. e. all of the above. 13. Which statement regarding pneumoconioses is incorrect? a. Particle size is an important determinant of where in the respiratory tract a particle will settle. b. Symptoms of a pneumoconiosis usually occur within days of exposure. c. Progressive massive fibrosis can be the outcome of a variety of pneumoconioses. d. Pneumoconioses includes cases of hypersensitivity pneumonitis and asthma caused by organic dusts. 14. Complications related to pneumoconioses include all of the following except: a. neoplasia. b. Caplan's syndrome. c. left-sided heart failure. d. pulmonary hypertension. 15. Asbestos a. is not normally found in human lungs. b. exposure is reflected by the presence of concentric nodules with birefringent material. c. is a highly soluble substance which is quickly digested and removed from the lung. d. is used in a variety of products employed in building construction.

18. Participants in the pathogenesis of pneumoconioses are thought to include a. macrophages. b. fibroblasts. c. fibrogenic factors. d. all of the above. 19. Which infectious disease occurs with significantly increased frequency in patients with silicosis? a. histoplasmosis. b. bacterial pneumonia. c. tuberculosis. d. viral pneumonia. 20. Histologic findings expected in silicosis include which two of the following? a. collagenous nodules with concentric laminations. b. crystals which are birefringent with polarization. c. necrotizing granulomas. d. ferruginous bodies. 21. An elderly coal miner complains of worsening dyspnea and a chronic cough productive of blackish sputum. His chest X ray shows multiple large (>2 cm.) densities in both lungs, particularly in the upper lobes. What would his lung biopsy be expected to show? a. collagenous nodules with concentric laminations b. large scars with numerous coal dust - containing macrophages c. granulomas d. mesothelioma

Obstructive lung diseases 1. Emphysema and chronic bronchitis a. are mutually exclusive disorders b. are pathologically defined disorders c. produce restrictive functional abnormalities d. are often referred to by the term chronic obstructive pulmonary disease 2. The mechanism by which the dilated air spaces of emphysema develop differs from that by which the dilated air spaces of "honeycomb lung" develop in that a. in honeycomb lung, air space walls are destroyed b. in emphysema, air space walls are destroyed c. in honeycomb lung, air trapping is the initiating event d. in emphysema, air trapping is the initiating event 3. Smoking is most closely associated with which type of emphysema? (choices below) 4. Which type of emphysema develops in alpha-1-antitrypsin deficiency? (choices below) a. centriacinar b. panacinar c. paraseptal d. irregular

16. The relative risk of developing lung carcinoma is highest in a(n)


a. asbestos-exposed nonsmoker. b. nonexposed smoker. c. asbestos-exposed smoker. d. nonexposed nonsmoker. 17. Histologic diagnosis of asbestosis requires which two of the following findings? a. asbestos bodies. b. separate densely collagenous nodules. c. granulomas. d. diffuse interstitial fibrosis.

5. Which statement regarding the pathogenesis of emphysema is incorrect? a. In smokers, increased elastase activity contributes to the development of emphysema. b. In smokers, neutrophil and macrophage accumulation in alveoli is important. c. A1AT function is inhibited by oxidants in smoke. d. Ciliary dysfunction is an important factor in the development of emphysema.

6. Which statement regarding chronic bronchitis is incorrect? a. Chronic bronchitis is defined as a persistent cough with sputum production for at least 3 mos. in at least two consecutive years. b. Cigarette smoking predisposes to chronic bronchitis by impairing ciliary function. c. In patients with chronic bronchitis, pigmented macrophages are often seen in bronchioles. d. Chronic bronchitis only develops in smokers. 7. Pathologic findings characteristic of chronic bronchitis include a. eosinophil infiltrates in bronchial walls b. increased size of the bronchial submucosal glands c. enlarged distal air spaces d. intraalveolar neutrophilic infiltrates 8. Which of the following statements about asthma is incorrect? a. Antigens triggering attacks of atopic asthma cause wheal-andflare reactions. b. A Th2 response is important in the pathogenesis of asthma. c. Viral and fungal infections are involved in the pathogenesis of some types of asthma. d. Reflex bronchodilation in asthma is initiated by release of inflammatory mediators. e. Airway fibrosis can result if asthma is not treated. 9. A 30 year old patient with an obstructed bronchus due to a tumor is most likely to develop which of the following diseases? a. emphysema b. chronic bronchitis c. asthma d. bronchiectasis 10. Factors predisposing to development of bronchiectasis include all of the following except: a. dysfunctional cilia b. necrotizing pneumonia c. emphysema d. cystic fibrosis 11. An asthmatic patient has peripheral eosinophilia (15% of WBC), neuropathy, and pulmonary infiltrates. A lung biopsy is most likely to show a. fungi colonizing bronchi b. honeycomb changes c. hyaline membranes d. necrotizing vasculitis

3. Factors associated with an increased likelihood of developing lung cancer include all of the following except a. radiation b. residence in a rural area c. previous lung cancer d. uranium mining Use choices a-d for questions 4 - 11. a. squamous cell carcinoma b. adenocarcinoma c. small cell carcinoma d. large cell carcinoma 4. What is the most common type of bronchogenic carcinoma? 5. Which type of lung carcinoma is most closely correlated with a smoking history and metastasizes later than the other types? 6. Which type of lung carcinoma tends to occur in a peripheral location? 7. Which is the most aggressive type of lung carcinoma? The following histologic findings are characteristic of which types of lung carcinoma? 8. glandular structures with an acinar pattern 9. small cells with high nuclear:cytoplasmic ratios and little cytoplasm 10. sheets of cells with occasional keratin pearls 12. large cells without keratinization or gland formation 13. Evidence for neuroendocrine differentiation includes a. intracellular mucin b. dense core granules c. intercellular bridges d. glycogen granules 14. A neuroendocrine tumor which is not related to smoking is the a. bronchioloalveolar carcinoma b. small cell carcinoma c. carcinoid d. giant cell carcinoma

15. Distant metastases from carcinomas of the lung are most likely to
be discovered in the a. adrenal glands b. bones c. brain d. ovaries

Tumors

1. Lung carcinoma
a. Rarely shows inactivation or deletion of p53 b. Has no recognizable precursor lesions c. is unrelated to K-RAS mutation d. Includes small cell and non-small cell carcinomas 2. The frequency of lung cancer is related to all of the following except: a. the number of cigarettes smoked per day b. the duration of smoking c. the tendency to inhale d. all of the above are related to the frequency of lung cancer

16. The overall 5 year survival rate for carcinoma of the lung is
closest to a. 0% b. 10% c. 25% d. 50%

17. Which of the following is least likely to be a manifestation of lung


carcinoma? a. Cushing's syndrome b. diabetes c. abscess d. Horner's syndrome

18. Which statement about bronchioloalveolar carcinoma is incorrect? a. This tumor may show Clara cell differentiation. b. It may mimic pneumonia on chest X ray. c. It is a type of adenocarcinoma. d. Its 5 year survival rate is lower than the average for bronchogenic carcinomas. 19. Which statement about the bronchial carcinoid is correct? a. This tumor is always benign. b. Intraluminal growth in a bronchus is typical. c. It usually develops in elderly people. d. Atypical carcinoids have less pleomorphism and fewer mitoses than typical carcinoids. 20. A feature which suggests that a tumor may be metastatic to the lung rather than primary is a. multiple nodules in multiple lobes of lung b. an irregularity in the mucosa of a bronchus c. tumor extension through the pleura d. development of SVC syndrome Atelectasis 1. Which pairing is correct? a. obstructive atelectasis --- aspirated peanut b. obstructive atelectasis --- pleural effusion c. compressive atelectasis --- asthma d. contraction atelectasis --- adult respiratory distress syndrome Pleural diseases

5. Which statement about pneumothoraces is correct? a. Tension pneumothoraces produce characteristic findings on CXR. b. Pneumothoraces do not produce respiratory distress. c. Superficial skin wounds to the thorax cause traumatic pneumothoraces. d. Pneumothorax is currently the preferred treatment for tuberculosis. 6. Malignant mesothelioma a. is the most common malignancy involving the pleural space b. is an indolent malignancy with an associated long life expectancy c. has a uniform histologic appearance, and does not usually require special modalities (special stains, electron microscopy) d. is associated with asbestos exposure, but occurs less often than bronchogenic carcinoma in people with a history of exposure

1. Transudative pleural effusions typically have all of the following


etiologies except: a. congestive heart failure b. nephrotic syndrome c. metastatic carcinoma d. cirrhosis of the liver 2. A milky-looking pleural fluid specimen has a protein of 4.0 mg/dL and LD of 250 IU. The most likely cause of the effusion is a. bronchogenic carcinoma b. viral pneumonia c. congestive heart failure d. pulmonary infarct 3. Elevated amylase activity in a pleural fluid specimen is most consistent with which of the following etiologies for the pleural effusion? a. rheumatoid disease b. ruptured esophagus c. tuberculosis d. pulmonary infarct 4. Which statement about empyemas is incorrect? a. Culture of the pleural fluid may reveal a bacterial pathogen. b. The predominant inflammatory cell in the exudate is the neutrophil. c. Empyemas develop more commonly by hematogenous spread than by extension from an adjacent pneumonia. d. A subdiaphragmatic infection can lead to development of am empyema.

ANSWERS
Vascular 1. d 2a. a b. c c. b d. d 3. c 4. d 5. b 6. a 7. a 8. c 9. b 10. c 11. b 12. a 13. b 14. d 15. a Bacterial 1. d 2. c 3. b 4. a 5. c 6. b 7. a 8. c 9. b 10. b 11. c 12. d Atypical Pneumonias 1. e 2. b 3. d 4. a 5. d 6. a 7. c 8. c 9. b Tuberculosis 1. d 2. c 3. a 4. d 5. a 6. b 7. c 8. c 9. a 10a. o b. n c. r d. m e. p f. t g. s h. q Fungal 1. c 2. a 3. e 4. c 5. d 6. b 7. c 8. c 9. b 10. a Interstitial 1. c 2. a 3. e 4. a 5. c 6. a 7. c 8. c 9. a 10. a 11. d 12. e 13. b 14. c 15. d 16. c 17. a,d 18. d 19. c 20. a,b 21. b Obstructive 1. d 2. b 3. a 4. b 5. d 6. d 7. b 8. d 9. d 10. c 11. d Tumors 1. d 2. d 3. b 4. a, b 5. a 6. b 7. c 8. b 9. c 10. a 11. missing 12. d 13. b 14. c 15. a 16. b 17. b 18. d 19. b 20. a Atelectasis 1. a Pleural 1. c 2. a 3. b 4. c 5. a 6. d

Das könnte Ihnen auch gefallen